6 tg pi x: Решение №1806 Найдите корень уравнения tg(π(x+6)/3)=√3.

Решутест. Продвинутый тренажёр тестов

Решил заданий

Не решил заданий

Осталось заданий

История решения
5278 — не приступал 8547 — не приступал 3560 — не приступал 3797 — не приступал 8657 — не приступал 5453 — не приступал 6734 — не приступал 7899 — не приступал 5642 — не приступал 5350 — не приступал

Формат ответа: цифра или несколько цифр, слово или несколько слов. Вопросы на соответствие «буква» — «цифра» должны записываться как несколько цифр. Между словами и цифрами не должно быть пробелов или других знаков.

Примеры ответов: 7 или здесьисейчас или 3514

Раскрыть Скрыть

№1

Решите уравнение $\cos\displaystyle\frac{\pi (2x-1)}{6} = \frac{\sqrt{3}}{2}$. В ответе запишите наибольший отрицательный корень.

№2

Решите уравнение $\sin\displaystyle\frac{\pi (2x-1)}{6} = \frac{\sqrt{3}}{2}$. В ответе запишите наибольший отрицательный корень.

№3

Решите уравнение $\sin\displaystyle\frac{\pi (8x-7)}{4} = 1$. В ответе запишите наименьший положительный корень.

№4

Решите уравнение $\sin\displaystyle\frac{\pi (x-3)}{4} = -\frac{\sqrt{2}}{2}$. В ответе запишите наименьший положительный корень.

№5

Найдите корень уравнения: $\cos\displaystyle\frac{\pi (x-7)}{3} = \frac{1}{2}$. В ответе запишите наибольший отрицательный корень.

№6

Решите уравнение $\sin\displaystyle\frac{\pi x}{3} = 0,5$.

В ответе напишите наименьший положительный корень.

№7

Решите уравнение $tg\displaystyle\frac{\pi x}{4} = -1$. В ответе напишите наибольший отрицательный корень.

№8

Решите уравнение $tg\displaystyle\frac{\pi(8x + 9)}{3} = -\sqrt{3}$. В ответе напишите наибольший отрицательный корень.

№9

Решите уравнение $tg\displaystyle\frac{\pi(x + 9)}{6} = \frac{1}{\sqrt{3}}$. В ответе напишите наибольший отрицательный корень.

№10

Решите уравнение $tg\displaystyle\frac{\pi(2x + 7)}{6} = \sqrt{3}$. В ответе напишите наибольший отрицательный корень.

решите уравнение tg(п+x)+2 tg(п/2+x)+1=0 — Знания.site

Ответы 6

мне математичка говорила так

  • Автор:

    acejtn8

  • Оценить ответ:

    0

уже не помню

темболее сначала решается по теореми виета откуда 2 взялась?

  • Автор:

    bud

  • Оценить ответ:

    0

корни 1 и 1 их можно не писать два раза

  • Автор:

    duke80

  • Оценить ответ:

    0

А ты покажи свои рассуждения учителю,вот она «обрадуется»!

tg(п+x)+2 tg(п/2+x)+1=0
tgx-2ctgx+1=0
tgx-2/tgx+1=0
tg²x+tgx-2=0    tgx≠0
tgx=a
a²+a-2=0
a1+a2=-1 U a1*a2=-2
a1=1⇒tgx=1⇒x=π/4+πn
a2=-2⇒tgx=-2⇒x=-arctg2+πn

Знаешь ответ? Добавь его сюда!

Последние вопросы

  • Литература

    4 часа назад

    Как сделать план

  • Геометрия

    6 часов назад

    ПОМОГИТЕ

  • Обществознание

    11 часов назад

    ужассссссссссссссссссс

    я из аккаунта этого вылетаю

  • Алгебра

    11 часов назад

    СРОЧНО!! Разложите на множители!

    3x3-5x2y+9x-15

  • Другие предметы

    12 часов назад

    Какая из библейских историй вам запомнилась?Чему учит каждая из них?

  • Литература

    13 часов назад

    Краткий пересказ повести мелентьева одни сутки войны

  • Математика

    14 часов назад

    Помогите пожалуйста сделать букву а

  • Математика

    17 часов назад

    Помогите пожалуйста сделать букву а

  • Другие предметы

    18 часов назад

    что делать если я бибизяна?

  • Математика

    19 часов назад

    Модно эту шлюху выебать с неграми жоско в рот .

  • Биология

    20 часов назад

    Васе надо выебать 245 волосатых мужиков. Ежедневно он ебёт на одно и то же количество мужиков больше по сравнению с предыдущим днём. Известно, что за первый день Вася выебал 11 мужиков в попки . Определите, сколько всего надо выебать Вася в последний день, если со всеми он справился за 1 дней.

  • Математика

    20 часов назад

    Васе надо выебать 245 волосатых мужиков. Ежедневно он ебёт на одно и то же количество мужиков больше по сравнению с предыдущим днём. Известно, что за первый день Вася выебал 11 мужиков в попки . Определите, сколько всего надо выебать Вася в последний день, если со всеми он справился за 1 дней.

  • Физика

    1 день назад

    Доброй ночи, помогите пожалуйста с заданием по физике, электростатика.

    Между зарядами -q и +9q расстояние равно 8 см. На каком расстоянии от первого заряда находится точка, в которой напряженность поля равна нулю?

  • Математика

    1 день назад

    В равнобедренной трапеции ABCD с основаниями AD и BC O-точка пересечения диагоналей, угол CAD равен 36°. Чему равен угол AOD? Ответ дайте в градусах

  • Алгебра

    1 день назад

    ПОМОГИТЕ РЕШИТЬ

3 6 Решить для ? cos(x)=1/2 7 Найти x sin(x)=-1/2 8 Преобразование градусов в радианы 225 9 Решить для ? cos(x)=(квадратный корень из 2)/2 10 Найти x cos(x)=(квадратный корень из 3)/2 11 Найти x sin(x)=(квадратный корень из 3)/2 92=9 14 Преобразование градусов в радианы 120 градусов 15 Преобразование градусов в радианы 180 16 Найти точное значение желтовато-коричневый(195) 92-4 38 Найти точное значение грех(255) 39 Оценить лог база 27 из 36 40 Преобразовать из радианов в градусы 2 шт. 92-3sin(x)+1=0 43 Найти x tan(x)+ квадратный корень из 3=0 44 Найти x sin(2x)+cos(x)=0 45 Упростить (1-cos(x))(1+cos(x)) 92=25 59 График f(x)=- натуральный логарифм x-1+3 60 Найдите значение с помощью единичного круга угловой синус(-1/2) 61 Найти домен квадратный корень из 36-4x^2 92=0 66 Найти x cos(2x)=(квадратный корень из 2)/2 67 График у=3 68 График f(x)=- логарифмическая база 3 x-1+3 92 71 Найти x квадратный корень из x+4+ квадратный корень из x-1=5 72 Решить для ? cos(2x)=-1/2 73 Найти x логарифмическая база x из 16=4 9х 75 Упростить (cos(x))/(1-sin(x))+(1-sin(x))/(cos(x)) 76 Упростить сек(х)sin(х) 77 Упростить кубический корень из 24 кубический корень из 18 92=0 96 Найти x 3x+2=(5x-11)/(8г) 97 Решить для ? sin(2x)=-1/2 98 Найти x (2x-1)/(x+2)=4/5 92+n-72)=1/(n+9)

Мэтуэй | Популярные задачи

92
1 Найти точное значение грех(30)
2 Найти точное значение грех(45)
3 Найти точное значение грех(30 градусов)
4 Найти точное значение грех(60 градусов)
5 Найти точное значение загар (30 градусов)
6 Найти точное значение угловой синус (-1)
7 Найти точное значение грех(пи/6)
8 Найти точное значение cos(pi/4)
9 Найти точное значение грех(45 градусов)
10 Найти точное значение грех(пи/3)
11 Найти точное значение арктан(-1)
12 Найти точное значение cos(45 градусов)
13 Найти точное значение cos(30 градусов)
14 Найти точное значение желтовато-коричневый(60)
15 Найти точное значение csc(45 градусов)
16 Найти точное значение загар (60 градусов)
17 Найти точное значение сек(30 градусов)
18 Найти точное значение cos(60 градусов)
19 Найти точное значение соз(150)
20 Найти точное значение грех(60)
21 Найти точное значение cos(pi/2)
22 Найти точное значение загар (45 градусов)
23 Найти точное значение arctan(- квадратный корень из 3)
24 Найти точное значение csc(60 градусов)
25 Найти точное значение сек(45 градусов)
26 Найти точное значение csc(30 градусов)
27 Найти точное значение грех(0)
28 Найти точное значение грех(120)
29 Найти точное значение соз(90)
30 Преобразовать из радианов в градусы пи/3
31 Найти точное значение желтовато-коричневый(30)
35 Преобразовать из радианов в градусы пи/6
36 Найти точное значение детская кроватка(30 градусов)
37 Найти точное значение арккос(-1)
38 Найти точное значение арктический(0)
39 Найти точное значение детская кроватка(60 градусов)
40 Преобразование градусов в радианы 30
41 Преобразовать из радианов в градусы (2 шт.

Сколько дней до 13 октября 2019: Сколько дней прошло с 13 октября 2019?

Сколько дней прошло с 13 октября 2019?

Калькулятор «Дней до даты»

Сколько дней до

Сколько времени прошло с 13 октября 2019?

Ответ: Прошло 3 года, 7 месяцев и 9 дней с


(сегодня (24 мая 2023) это 3 года, 7 месяцев и 1 неделю после 13 октября 2019)

это также

  • 3,611 Года
  • или
  • 43,355 Месяца
  • или
  • 188,429 Недель
  • или
  • 1 319 Дней
  • или
  • 31 656 Часов
  • или
  • 1 899 360 Минут
  • или
  • 113 961 600 Секунд
  • или
  • 3 года, 7 месяцев и 9 дней

13 октября 2019 — Отсчет времени

Временная шкала

24 мая 2023

3.61 года

13 октября 2019

43. 35 месяца

Информация о дне: 13 октября 2019

  • 13 октября 2019 это воскресенье (Выходной день)
  • Этот день находится на 41 (сорок первой) неделе 2019 года
  • Это 286 (двести восемьдесят шестой) День в году
  • До конца 2019 года остается 79 дней (год завершен на 78.36%)
  • Это 43 (сорок третий) день Осени 2019
  • 2019 это не Високосный Год (365 Дней)
  • Кол-во дней в Октябре 2019: 31
  • Знак Зодиака для дня 13 октября 2019 это Весы (libra)
  • Возраст человека, родившегося 13 октября 2019 составляет 3.61 лет
  • 13 октября 2019 как Unix Timestamp: 1570924800

Календарь на Октябрь 2019

Поделитесь текущим расчетом

Печать

https://calculat.io/ru/date/how-many-until/13-october-2019

<a href=»https://calculat. io/ru/date/how-many-until/13-october-2019″>Сколько дней прошло с 13 октября 2019? — Calculatio</a>

О калькуляторе «Дней до даты»

Онлайн калькулятор времени до даты поможет узнать сколько времени осталось до заданной даты. Например, легко узнать сколько времени осталось до вашего Дня Рождения. Также, можно узнать сколько времени прошло с заданной даты. Например, он может помочь узнать сколько времени прошло с 13 октября 2019? Выберите нужную дату, (например ’13 октября 2019′) и нажмите кнопку ‘Посчитать’.

Калькулятор «Дней до даты»

Сколько дней до

Таблица конвертации

ДатаВремя до даты
29 сентября 2019-3 года, 7 месяцев и 23 дня
30 сентября 2019-3 года, 7 месяцев и 22 дня
01 октября 2019-3 года, 7 месяцев и 21 день
02 октября 2019-3 года, 7 месяцев и 20 дней
03 октября 2019-3 года, 7 месяцев и 19 дней
04 октября 2019-3 года, 7 месяцев и 18 дней
05 октября 2019-3 года, 7 месяцев и 17 дней
06 октября 2019-3 года, 7 месяцев и 16 дней
07 октября 2019-3 года, 7 месяцев и 15 дней
08 октября 2019-3 года, 7 месяцев и 14 дней
09 октября 2019-3 года, 7 месяцев и 13 дней
10 октября 2019-3 года, 7 месяцев и 12 дней
11 октября 2019-3 года, 7 месяцев и 11 дней
12 октября 2019-3 года, 7 месяцев и 10 дней
13 октября 2019-3 года, 7 месяцев и 9 дней
14 октября 2019-3 года, 7 месяцев и 8 дней
15 октября 2019-3 года, 7 месяцев и 7 дней
16 октября 2019-3 года, 7 месяцев и 6 дней
17 октября 2019-3 года, 7 месяцев и 5 дней
18 октября 2019-3 года, 7 месяцев и 4 дня
19 октября 2019-3 года, 7 месяцев и 3 дня
20 октября 2019-3 года, 7 месяцев и 2 дня
21 октября 2019-3 года, 7 месяцев и 1 день
22 октября 2019-3 года и 7 месяцев
23 октября 2019-3 года, 6 месяцев и 30 дней
24 октября 2019-3 года, 6 месяцев и 29 дней
25 октября 2019-3 года, 6 месяцев и 28 дней
26 октября 2019-3 года, 6 месяцев и 27 дней
27 октября 2019-3 года, 6 месяцев и 26 дней
28 октября 2019-3 года, 6 месяцев и 25 дней

Сколько времени прошло с 13 октября 2019 года до сегодняшнего дня

Калькулятор делает расчет: сколько дней, лет, часов, минут или секунд прошло с 13. 10.2019 по текущий момент времени.

Сколько прошло с 13 октября 2019 года?

В общем

  • 3 года
  • 7 месяцев
  • 11 дней
  • 15 часов
  • 26 минут
  • 4 секунды

В целых величинах

  • 3 года
  • 43 месяца
  • 188 недель
  • 1319 дней
  • 31671 час
  • 1900286 минут
  • 114017164 секунды

Часовой пояс: Europe/Berlin

Сегодняшняя дата

24 мая 2023 года

Информация о дне: 13 октября 2019 года

  • 13 октября 2019 года — это Воскресенье (Выходной день)
  • Дней в октябре: 31
  • 2019 — это не Високосный год (365 дней)
  • Октябрь: пора года — Осень
  • Знак зодиака 13 октября 2019 года — Весы (libra)
  • Unix Timestamp: 1570924800

Календарь на октябрь 2019 года

Ближайшие даты

07. 10.2019

08.10.2019

09.10.2019

10.10.2019

11.10.2019

12.10.2019

14.10.2019

15.10.2019

16.10.2019

17.10.2019

18.10.2019

19.10.2019

Все даты 2019 года

Другие даты

13.10.201918.05.2023

13.10.201919.05.2023

13.10.201920.05.2023

13.10.201921.05.2023

13.10.201922.05.2023

13.10.201923.05.2023

13.10.201924.05.2023

13.10.201925.05.2023

13.10.201926.05.2023

13. 10.201927.05.2023

13.10.201928.05.2023

13.10.201929.05.2023

Вопросы и ответы

Данный инструмент является калькулятором дат. Он вычисляет количество времени, которое прошло с заданной даты на текущий момент. Результат можно отобразить в разных единицах измерения.

Просто выберите нужную вам дату с помощью календаря и сразу получайте результат!

Нет, калькулятор предназначен только для подсчета прошедшего времени от указанной даты до текущей даты. Для будущих дат используйте другой калькулятор, который есть на нашем сайте!

Рекомендуем посмотреть

Спасибо за обратную связь!

Если потребуется, мы вышлем ответ на указанную почту.

×

Обратная связь

Оставьте сообщение и мы обязательно вам ответим!

Сообщение *

Имя

E-mail *

Поддержите нас!

Мы рады, что вы пользуетесь нашим сервисом!
Чтобы отблагодарить нас за бесплатные инструменты — отключите блокировщик рекламы на сайте или сделайте пожертвование! Это очень поможет развитию наших проектов!
Спасибо 🙂

99₽

99₽

199₽

199₽

499₽

499₽

Любая сумма

Пожертвовать
  • Ether: 0x2764e55bbbc6e60fa0678da98aae46635e850bdc
  • Bitcoin cash: qzm2pkf9sdzc0lpe39lgh52u2gc52majqcnxc0uz8j

Сколько дней назад было 13 октября 2019 года?

Калькулятор «Дней до даты»

Сколько дней осталось до

Сколько времени осталось с 13 октября 2019 года?

Ответ: Было 3 года, 7 месяцев и 9 дней с


( Сегодня (24, 20 мая) 23) 3 года, 7 месяцев и 1 неделя после 13 октября 2019 )

Он же

  • 3,611 Год
  • или
  • 43. 355 Месяцы
  • или
  • 188.429 Недели
  • или
  • 9001 1 1 319 Дни
  • или
  • 31 656 Часы
  • или
  • 1 899 360 Минуты
  • 9002 8 или
  • 113 961 600 Секунды
  • или
  • 3 года, 7 месяцев и 9 дней

13 октября 2019 г. — Обратный отсчет

900 03

Хронология

24 мая 2023 г.

3,61 года

13 октября 2019 г.

43,35 месяца

О дне: 13 октября , 2019

  • 13 октября 2019 выпадает на воскресенье (Выходные)
  • Этот день 42-й (сорок второй) Неделя 2019 года
  • Это 286-й (двести восемьдесят шестой) День года
  • Есть 79 дней осталось до конец 2019 года
  • 13 октября 2019 года 78,36% года завершено
  • Это 43-й (сорок третий) День осени 2019
  • 2019 год не високосный год (365 дней)
  • Количество дней в октябре 2019 года : 31
  • Знак Зодиака от 13 октября 2019 года: Весы (весы)
  • Человек, родившийся 13 октября 2019 года, будет 3,61 лет 90 012
  • 13 октября 2019 г. as временная метка Unix 9

    Поделиться этим расчетом Сколько дней назад было 13 октября 2019 года? — Расчет

    О калькуляторе «Дней до даты»

    Этот онлайн-калькулятор дат поможет вам рассчитать, сколько дней осталось до определенной даты. Например, легко узнать, сколько времени осталось до дня рождения. Вы также можете узнать, сколько времени прошло с определенной даты. Например, это может помочь вам узнать, сколько времени прошло с 13 октября 2019 г.? Выберите нужную дату (например, «13 октября 2019 г.») и нажмите кнопку «Рассчитать».

    Калькулятор «Дней до даты»

    Сколько дней осталось до

    Таблица преобразования

    90 179 90 179
    Дата Текущая дата
    29 сентября 2019 г. -3 года, 7 месяцев и 23 дня
    30 сентября 2019 г. -3 года, 7 месяцев и 22 дня
    01 октября 2019 г. -3 года, 7 месяцев и 21 день
    02 октября 2019 г. -3 года, 7 месяцев и 20 дней
    03 октября 2019 г. -3 года, 7 месяцев и 19 дней
    04 октября 2019 г. -3 года, 7 месяцев и 18 дней
    05 октября 2019 г. -3 года, 7 месяцев и 1 7 дней
    06 октября, 2019 -3 года, 7 месяцев и 16 дней
    07 октября 2019 г. -3 года, 7 месяцев и 15 дней
    08 октября 2019 г. -3 года, 7 месяцев и 14 дней
    09 октября 2019 г. -3 лет, 7 месяцев и 13 дней
    10 октября 2019 г. -3 года, 7 месяцев и 12 дней
    11 октября 2019 г. -3 года, 7 месяцев и 11 дней
    Октябрь 12, 2019 -3 года, 7 месяцев и 10 дней
    13 октября 2019 г. -3 года, 7 месяцев и 9 дней
    14 октября 2019 г. -3 года, 7 месяцев и 8 дней
    15.10.2019 -3 лет, 7 месяцев и 7 дней
    16 октября 2019 г. -3 года, 7 месяцев и 6 дней
    17 октября 2019 г. -3 года, 7 месяцев и 5 дней
    Октябрь 18, 2019 -3 года, 7 месяцев и 4 дня
    19 октября 2019 г. -3 года, 7 месяцев и 3 дня
    20 октября 2019 г. -3 года, 7 месяцев и 2 дня
    21.10.2019 -3 лет, 7 месяцев и 1 день
    22 октября 2019 года -3 года и 7 месяцев
    23 октября 2019 года -3 года, 6 месяцев и 30 дней 9 0184
    24 октября 2019 г. -3 года, 6 месяцев и 29дней
    25 октября 2019 г. -3 года, 6 месяцев и 28 дней
    26 октября 2019 г. -3 года, 6 месяцев и 27 дней 9018 4
    27 октября 2019 г. — 3 года, 6 месяцев и 26 дней
    28 октября 2019 г. -3 года, 6 месяцев и 25 дней

    Сколько дней до 13 октября?

    Подсчитать, сколько дней осталось до 13 октября



    13 октября 2023 г. — 142 дня с сегодня


    Сколько осталось до 13 октября?

    С сегодняшнего дня до 13 октября осталось 142 дня. Это означает, что до этого момента осталось 20,29 недель, 3408,0 часов и 5,07 месяцев. Мы используем этот расчет довольно часто в календаре, даже если мы этого не осознаем. Обратный отсчет чьего-то дня рождения, юбилея или особой даты важен, чтобы вовремя заказать подарки! Если 13 октября имеет для вас особое значение, сделайте одолжение себе в будущем и установите напоминание в календаре на день раньше и сделай это повторяющимся. Пожалуйста.

    Обратный отсчет до 13 октября

    Дней до 13 октября?

    142 дня

    Недели до 13 октября?

    20,29 недели

    Часов до 13 октября?

    3408,0 часов

    Месяцев до 13 октября?

    5,07 месяцев

    Сколько минут до 13 октября

    204480 минут

    Сколько секунд до 13 октября

    1 2268800 секунд

    Сколько лет до 13 октября

    0,39 лет

    13 октября составляет 78% в течение года

    78%

    Сколько бизнес дней до 13 октября?

    До 13 октября осталось 102 рабочих дня.

    В деловом мире время до определенной даты совершенно другое. Десять рабочих дней составляют две календарные недели. и один месяц составляет всего двадцать дней производства. Это меняет то, сколько времени корпорация отрабатывает традиционный 9-5 система подсчета времени реально можно потратить на проекты или работу. Это может добавить слой сложность на расчеты времени.

    Чрезмерное упрощение расчета рабочих дней до 13 октября заключается в подсчете общего количества дней 142 и вычитании общего количества выходных.

    Самый простой способ настроить разницу во времени? Используйте калькулятор даты и времени, подобный этому, и мгновенно получите отвечать.

    В период с 13 октября средний человек потратил…

    • 30501,6 часа Сон
    • 4055,52 часов Еда и питье
    • 6645,6 часов Домашняя деятельность
    • 1976,64 часа Работа по дому
    • 2181,12 часа Приготовление пищи и уборка
    • 681,6 часа Уход за газоном и садом
    • 11928,0 часов Работа и связанная с работой деятельность
    • 10973,76 часов Рабочий
    • 17960,16 часов Отдых и спорт
    • 9746,88 часов Просмотр телевизора

    13 октября Статистика:

    • В этом году 13 октября — пятница
    • В следующем году 13 октября будет суббота
    • День недели: Пятница
    • День года: 286
    • День месяца: 13

    В пятницу, 13 октября, было 286, что составляет 78% до 2023 года.

Мэтуэй | Популярные задачи

92) 9(3x) по отношению к x 92+1
1 Найти производную — d/dx бревно натуральное х
2 Оценить интеграл интеграл натурального логарифма x относительно x
3 Найти производную — d/dx
21 Оценить интеграл интеграл от 0 до 1 кубического корня из 1+7x относительно x
22 Найти производную — d/dx грех(2x)
23 Найти производную — d/dx
41 Оценить интеграл интеграл от cos(2x) относительно x
42 Найти производную — d/dx 1/(корень квадратный из х)
43 Оценка интеграла 9бесконечность
45 Найти производную — d/dx х/2
46 Найти производную — d/dx -cos(x)
47 Найти производную — d/dx грех(3x)
68 Оценить интеграл интеграл от sin(x) по x
69 Найти производную — d/dx угловой синус(х)
70 Оценить предел ограничение, когда x приближается к 0 из (sin(x))/x 92 по отношению к х
85 Найти производную — d/dx лог х
86 Найти производную — d/dx арктан(х)
87 Найти производную — d/dx бревно натуральное 5х92

Интегрирование квадратного корня из x | Интеграл от 1 по корню x

от promath

Присоединяйтесь к нашему каналу Telegram

Интеграция корня x равна 2/3 x 3/2 +C. В этом посте мы вычислим интегрирование квадратного корня из x. Обратите внимание, что квадратный корень из x является алгебраической функцией. Мы будем использовать следующее степенное правило производных: 9{n+1}}{n+1}+c$ $ \cdots (i)$

, где $c$ — константа интегрирования.

Примечание: Для получения более подробной информации о квадратных корнях, пожалуйста, нажмите на страницу Квадратный корень из x: определение, символ, график, свойства, производная, интегрирование.

Что такое интеграция root x? Давай выясним.

Интегрирование корня x

Вопрос: Найти интегрирование корня x, то есть

найти $\int \sqrt{x} dx$

Ответ: 9{\frac{1}{2}}+c$

$=\dfrac{2}{3} x\sqrt{x}+c$

Таким образом, интегрирование корня x равно $\dfrac{2 }{3} х\sqrt{х}$.

Преобразовать презентацию в jpg: Конвертировать PPT в JPG онлайн — Convertio

Онлайн-конвертер PPT в JPG | Бесплатные приложения GroupDocs

Вы также можете конвертировать PPT во многие другие форматы файлов. Пожалуйста, смотрите полный список ниже.

PPT TO DOC Конвертер (Документ Microsoft Word)

PPT TO DOCM Конвертер (Документ Microsoft Word с поддержкой макросов)

PPT TO DOCX Конвертер (Документ Microsoft Word с открытым XML)

PPT TO DOT Конвертер (Шаблон документа Microsoft Word)

PPT TO DOTM Конвертер (Шаблон Microsoft Word с поддержкой макросов)

PPT TO DOTX Конвертер (Шаблон документа Word Open XML)

PPT TO RTF Конвертер (Расширенный текстовый формат файла)

PPT TO ODT Конвертер (Открыть текст документа)

PPT TO OTT Конвертер (Открыть шаблон документа)

PPT TO TXT Конвертер (Формат обычного текстового файла)

PPT TO MD Конвертер (Уценка)

PPT TO XPS Конвертер (Спецификация документа Open XML)

PPT TO TEX Конвертер (Исходный документ LaTeX)

PPT TO PDF Конвертер (Портативный документ)

PPT TO PS Конвертер (Постскриптум (PS))

PPT TO PCL Конвертер (Документ языка управления принтером)

PPT TO EPUB Конвертер (Формат файла цифровой электронной книги)

PPT TO MOBI Конвертер (Электронная книга Mobipocket)

PPT TO AZW3 Конвертер (Kindle eBook format)

PPT TO TIFF Конвертер (Формат файла изображения с тегами)

PPT TO TIF Конвертер (Формат файла изображения с тегами)

PPT TO JPEG Конвертер (Изображение в формате JPEG)

Преобразовать PPT TO PNG (Портативная сетевая графика)

Преобразовать PPT TO GIF (Графический файл формата обмена)

Преобразовать PPT TO BMP (Формат растрового файла)

Преобразовать PPT TO ICO (Файл значка Майкрософт)

Преобразовать PPT TO PSD (Документ Adobe Photoshop)

Преобразовать PPT TO WMF (Метафайл Windows)

Преобразовать PPT TO EMF (Расширенный формат метафайла)

Преобразовать PPT TO DCM (DICOM-изображение)

Преобразовать PPT TO DICOM (Цифровая визуализация и коммуникации в медицине)

Преобразовать PPT TO WEBP (Формат файла растрового веб-изображения)

Преобразовать PPT TO JP2 (Основной файл изображения JPEG 2000)

Преобразовать PPT TO EMZ (Расширенный сжатый метафайл Windows)

Преобразовать PPT TO WMZ (Метафайл Windows сжат)

Преобразовать PPT TO SVGZ (Сжатый файл масштабируемой векторной графики)

Преобразовать PPT TO TGA (Тарга Графика)

Преобразовать PPT TO PSB (Файл изображения Adobe Photoshop)

Преобразовать PPT TO XLS (Формат двоичного файла Microsoft Excel)

Преобразовать PPT TO XLSX (Электронная таблица Microsoft Excel Open XML)

Преобразовать PPT TO XLSM (Электронная таблица Microsoft Excel с поддержкой макросов)

Преобразовать PPT TO XLSB (Двоичный файл электронной таблицы Microsoft Excel)

Преобразовать PPT TO ODS (Открыть электронную таблицу документов)

Преобразовать PPT TO XLTX (Открытый XML-шаблон Microsoft Excel)

PPT TO XLT Преобразование (Шаблон Microsoft Excel)

PPT TO XLTM Преобразование (Шаблон Microsoft Excel с поддержкой макросов)

PPT TO TSV Преобразование (Файл значений, разделенных табуляцией)

PPT TO XLAM Преобразование (Надстройка Microsoft Excel с поддержкой макросов)

PPT TO CSV Преобразование (Файл значений, разделенных запятыми)

PPT TO FODS Преобразование (Плоская XML-таблица OpenDocument)

PPT TO SXC Преобразование (Электронная таблица StarOffice Calc)

PPT TO SVG Преобразование (Файл масштабируемой векторной графики)

PPT TO HTM Преобразование (Файл языка гипертекстовой разметки)

PPT TO HTML Преобразование (Язык гипертекстовой разметки)

PPT TO MHTML Преобразование (MIME-инкапсуляция совокупного HTML)

PPT TO MHT Преобразование (MIME-инкапсуляция совокупного HTML)

PPT TO PPT Преобразование (Презентация PowerPoint)

PPT TO PPS Преобразование (Слайд-шоу Microsoft PowerPoint)

PPT TO PPTX Преобразование (Презентация PowerPoint Open XML)

PPT TO PPSX Преобразование (Слайд-шоу PowerPoint Open XML)

PPT TO ODP Преобразование (Формат файла презентации OpenDocument)

PPT TO OTP Преобразование (Шаблон графика происхождения)

PPT TO POTX Преобразование (Открытый XML-шаблон Microsoft PowerPoint)

PPT TO POT Преобразование (Шаблон PowerPoint)

PPT TO POTM Преобразование (Шаблон Microsoft PowerPoint)

PPT TO PPTM Преобразование (Презентация Microsoft PowerPoint)

PPT TO PPSM Преобразование (Слайд-шоу Microsoft PowerPoint)

PPT TO FODP Преобразование (Плоская XML-презентация OpenDocument)

Онлайн-конвертер PPT в JPEG | Бесплатные приложения GroupDocs

Вы также можете конвертировать PPT во многие другие форматы файлов. Пожалуйста, смотрите полный список ниже.

PPT TO DOC Конвертер (Документ Microsoft Word)

PPT TO DOCM Конвертер (Документ Microsoft Word с поддержкой макросов)

PPT TO DOCX Конвертер (Документ Microsoft Word с открытым XML)

PPT TO DOT Конвертер (Шаблон документа Microsoft Word)

PPT TO DOTM Конвертер (Шаблон Microsoft Word с поддержкой макросов)

PPT TO DOTX Конвертер (Шаблон документа Word Open XML)

PPT TO RTF Конвертер (Расширенный текстовый формат файла)

PPT TO ODT Конвертер (Открыть текст документа)

PPT TO OTT Конвертер (Открыть шаблон документа)

PPT TO TXT Конвертер (Формат обычного текстового файла)

PPT TO MD Конвертер (Уценка)

PPT TO XPS Конвертер (Спецификация документа Open XML)

PPT TO TEX Конвертер (Исходный документ LaTeX)

PPT TO PDF Конвертер (Портативный документ)

PPT TO PS Конвертер (Постскриптум (PS))

PPT TO PCL Конвертер (Документ языка управления принтером)

PPT TO EPUB Конвертер (Формат файла цифровой электронной книги)

PPT TO MOBI Конвертер (Электронная книга Mobipocket)

PPT TO AZW3 Конвертер (Kindle eBook format)

PPT TO TIFF Конвертер (Формат файла изображения с тегами)

PPT TO TIF Конвертер (Формат файла изображения с тегами)

PPT TO JPG Конвертер (Файл изображения Объединенной группы экспертов по фотографии)

Преобразовать PPT TO PNG (Портативная сетевая графика)

Преобразовать PPT TO GIF (Графический файл формата обмена)

Преобразовать PPT TO BMP (Формат растрового файла)

Преобразовать PPT TO ICO (Файл значка Майкрософт)

Преобразовать PPT TO PSD (Документ Adobe Photoshop)

Преобразовать PPT TO WMF (Метафайл Windows)

Преобразовать PPT TO EMF (Расширенный формат метафайла)

Преобразовать PPT TO DCM (DICOM-изображение)

Преобразовать PPT TO DICOM (Цифровая визуализация и коммуникации в медицине)

Преобразовать PPT TO WEBP (Формат файла растрового веб-изображения)

Преобразовать PPT TO JP2 (Основной файл изображения JPEG 2000)

Преобразовать PPT TO EMZ (Расширенный сжатый метафайл Windows)

Преобразовать PPT TO WMZ (Метафайл Windows сжат)

Преобразовать PPT TO SVGZ (Сжатый файл масштабируемой векторной графики)

Преобразовать PPT TO TGA (Тарга Графика)

Преобразовать PPT TO PSB (Файл изображения Adobe Photoshop)

Преобразовать PPT TO XLS (Формат двоичного файла Microsoft Excel)

Преобразовать PPT TO XLSX (Электронная таблица Microsoft Excel Open XML)

Преобразовать PPT TO XLSM (Электронная таблица Microsoft Excel с поддержкой макросов)

Преобразовать PPT TO XLSB (Двоичный файл электронной таблицы Microsoft Excel)

Преобразовать PPT TO ODS (Открыть электронную таблицу документов)

Преобразовать PPT TO XLTX (Открытый XML-шаблон Microsoft Excel)

PPT TO XLT Преобразование (Шаблон Microsoft Excel)

PPT TO XLTM Преобразование (Шаблон Microsoft Excel с поддержкой макросов)

PPT TO TSV Преобразование (Файл значений, разделенных табуляцией)

PPT TO XLAM Преобразование (Надстройка Microsoft Excel с поддержкой макросов)

PPT TO CSV Преобразование (Файл значений, разделенных запятыми)

PPT TO FODS Преобразование (Плоская XML-таблица OpenDocument)

PPT TO SXC Преобразование (Электронная таблица StarOffice Calc)

PPT TO SVG Преобразование (Файл масштабируемой векторной графики)

PPT TO HTM Преобразование (Файл языка гипертекстовой разметки)

PPT TO HTML Преобразование (Язык гипертекстовой разметки)

PPT TO MHTML Преобразование (MIME-инкапсуляция совокупного HTML)

PPT TO MHT Преобразование (MIME-инкапсуляция совокупного HTML)

PPT TO PPT Преобразование (Презентация PowerPoint)

PPT TO PPS Преобразование (Слайд-шоу Microsoft PowerPoint)

PPT TO PPTX Преобразование (Презентация PowerPoint Open XML)

PPT TO PPSX Преобразование (Слайд-шоу PowerPoint Open XML)

PPT TO ODP Преобразование (Формат файла презентации OpenDocument)

PPT TO OTP Преобразование (Шаблон графика происхождения)

PPT TO POTX Преобразование (Открытый XML-шаблон Microsoft PowerPoint)

PPT TO POT Преобразование (Шаблон PowerPoint)

PPT TO POTM Преобразование (Шаблон Microsoft PowerPoint)

PPT TO PPTM Преобразование (Презентация Microsoft PowerPoint)

PPT TO PPSM Преобразование (Слайд-шоу Microsoft PowerPoint)

PPT TO FODP Преобразование (Плоская XML-презентация OpenDocument)

PPT в JPG — online-convert.

com

Лучшее качество
Нет визуальной разницы

Самый маленький файл
Маленький размер файла

Лучшее сжатие

Качество:

Определите качество получаемого изображения. Чем лучше качество, тем больше размер файла. Таким образом, более низкое качество также уменьшит размер файла.

Наилучшее сжатиеНаилучшее качество

0%

20%

40%

60%

80%

100%

Размер изменения:

Ширина:

пикс.

Высота:

пикс.

Применить цветовой фильтр: без измененийОттенки серогоМонохромныйОтменить цветаРетроСепия

Улучшение Резкость Сглаживание Удаление пятен Уравнять Нормализовать Устранение перекоса Нет многослойного

точек на дюйм: «/>

точек на дюйм

Обрезать пиксели из:

Верх:

пикс.

Низ:

пикс.

Слева:

пикс.

Справа:

пикс.

Установка черно-белого порога:

Укажите единицу разрешения для DPI: . нетдюймовCM

Установите используемый метод подвыборки цветности: без изменений4:4:44:4:04:2:24:2:04:1:14:1:0

Информация: Пожалуйста, включите JavaScript для корректной работы сайта.

  1. Выберите файл PPT , который вы хотите преобразовать
  2. Изменить качество или размер (необязательно)
  3. Нажмите «Начать преобразование», чтобы преобразовать файл из PPT в JPG
  4. Загрузите файл JPG

Как преобразовать PowerPoint в JPG

PowerPoint определенно лучший выбор, когда вам нужно сделать презентацию. Но когда вам нужно его представить, могут возникнуть некоторые проблемы. Например, на устройстве, которое вы хотите использовать для демонстрации слайдов, может не быть установлен PowerPoint. Кроме того, если вам нужно загрузить презентацию в облако, она может выглядеть не совсем так. Чтобы избежать возможных проблем, вам следует научиться конвертировать PowerPoint в JPG.

Когда вы превращаете слайды в изображения, вы можете просматривать их на любом устройстве и легко делиться ими. С форматом JPG намного проще работать, особенно если вам нужно передать файлы или поделиться ими с другими устройствами. В следующем руководстве вы узнаете, как сохранить PowerPoint в формате JPEG и устранить все возможные риски потери макета ваших слайдов.

Если вы решите сохранить слайды как изображения, сразу после завершения презентации вам нужно сделать следующее:

Перейдите в меню Файл и затем нажмите Сохранить как . Выберите место, где вы сохраните файл. Введите имя файла и из списка форматов файлов выберите JPEG и нажмите Сохранить .

Если в вашей презентации несколько слайдов, вы получите более одного изображения. Появится окно с вопросом « Какие слайды вы хотите экспортировать ” и вы можете экспортировать только текущий слайд или все.

Этот способ преобразования презентации в изображение удобен, если вы в настоящее время работаете в PowerPoint или на компьютере, на котором установлена ​​программа PowerPoint. Однако, если вы оказались вдали от настольного компьютера и вам нужно преобразовать презентацию PowerPoint в JPG, есть альтернативный способ сделать это.

Если у вас сейчас нет PowerPoint, вы все равно можете превратить слайды презентации в изображения. Вы можете сделать это на телефоне, планшете, ноутбуке или любом другом устройстве. Даже если у вас нет с собой презентации, вы можете попросить кого-нибудь прислать ее и сделать следующее:

Необходимое время: 5 минут.

Преобразование PowerPoint в JPG без PowerPoint за 4 простых шага

  1. Преобразование PowerPoint в PDF

    Перейдите на домашнюю страницу PDF Converter и выберите инструмент PowerPoint в PDF. Загрузите презентацию PowerPoint туда. Этот инструмент можно использовать бесплатно, электронная почта не требуется.

Y arctg x график функции: Функция y = arctgx — урок. Алгебра, 11 класс.

404 — Страница не найдена

Страницы

Партнеры сайта

_________________________________


404: Запрошенная страница с адресом [http://primer. by/algebra/funkcii/funkcija-yarctgx] не найдена.

Если Вы уверены, что набрали ссылку корректно, напишите, пожалуйста, об этом на:

меню пользователя

Новости


30.11.16 


17.03.15 


25.03.14 


29.08.13 


05. 05.13 



primer. by 2013-2016

Элементарная математика

  

Сканави М.И. Элементарная математика. 2-е изд., перераб. и доп., М.: 1974г. — 592с.

Книга представляет собой повторительный курс элементарной математики и рассчитана на тех, кто хочет пополнить, укрепить и систематизировать свои знания. Как и в первом издании, содержание ориентировано на программы вступительных экзаменов в технические вузы и, в особенности, на программы подготовительных отделений при высших учебных заведениях, для учащихся которых, как мы надеемся, книга окажется полезной.

(Книга включает в себя Ч1 — Арифметика, алгебра и элементарные функции и Ч2 — Геометрия. Каждый раздел включает в себя теоретическую часть и большое количество задач с решениями.)



Оглавление

ВВЕДЕНИЕ
Часть первая. АРИФМЕТИКА, АЛГЕБРА И ЭЛЕМЕНТАРНЫЕ ФУНКЦИИ
Глава I. ДЕЙСТВИТЕЛЬНЫЕ И КОМПЛЕКСНЫЕ ЧИСЛА
2. Простые и составные числа. Признаки делимости.
3. Наибольший общий делитель и наименьшее общее кратное.
4. Целые числа. Рациональные числа.
5. Десятичные дроби. Представление рациональных чисел десятичными дробями.
6. Иррациональные числа. Действительные числа.
7. Действия с приближенными числами.
8. Числовая ось. Координаты точки на плоскости.
§ 2. Степени и корни
9. Степени с натуральными показателями.
10. Степени с целыми показателями.
11. Корни.
12. Степени с рациональными показателями. Степени с действительными показателями.
13. Алгоритм извлечения квадратного корня.
§ 3. Комплексные числа
14. Основные понятия и определения.
15. Рациональные действия с комплексными числами.
16. Геометрическое изображение комплексных чисел. Тригонометрическая форма комплексного числа.
17. Действия с комплексными числами, заданными в тригонометрической форме. Формула Муавра.
18. Извлечение корня из комплексного числа.
Глава II. ТОЖДЕСТВЕННЫЕ ПРЕОБРАЗОВАНИЯ
19. Алгебраические выражения. Одночлены и многочлены.
20. Формулы сокращенного умножения.
21. Бином Ньютона.
22. Разложение многочлена на множители.
23. Дробные алгебраические выражения.
§ 2. Иррациональные алгебраические выражения
24. Радикалы из алгебраических выражений.
25. Освобождение от иррациональности в знаменателе дроби.
Глава III. ЛОГАРИФМЫ
26. Определение и свойства логарифмов.
27. Логарифмы по различным основаниям. Модуль перехода.
§ 2. Десятичные логарифмы
28. Характеристика и мантисса десятичного логарифма.
29. Применение десятичных логарифмов к вычислениям. n.
41. Обратная пропорциональная зависимость. Степенная функция с рациональным показателем степени.
42. Показательная функция.
43. Логарифмическая функция.
§ 3. Преобразование графиков
44. Параллельный сдвиг графика.
45. График квадратного трех члена.
46. График дробно-линейной функции.
47. Преобразование симметрии. Сжатие и растяжение графика.
48. Построение графиков функций.
49. Сложение графиков.
§ 4. Некоторые сведения о рациональных функциях
50. Целые и дробные рациональные функции. Деление многочленов.
51. Схема Горнера. Теорема Безу.
52. Нули многочлена. Разложение многочлена на множители.
Глава V. УРАВНЕНИЯ
53. Уравнение. Корни уравнения.
54. Равносильные уравнения.
55. Системы уравнений.
56. Графическое решение уравнений.
§. 2. Алгебраические уравнения с одной неизвестной
57. Число и кратность корней.
58. Уравнения первой степени (линейные уравнения).
59. Уравнения второй степени (квадратные уравнения).
60. Формулы Виета. Разложение квадратного трехчлена на множители.
61. Исследование квадратного уравнения.
62. Уравнения высших степеней. Целые корни.
63. Двучленные уравнения.
64. Уравнения, сводящиеся к квадратным.
65. Возвратные уравнения.
§ 3. Системы алгебраических уравнений
66. Линейные системы.
67. Определители второго порядка. Исследование линейных систем двух уравнений с двумя неизвестными.
68. Системы, состоящие из уравнения второй степени и линейного уравнения.
69. Примеры систем двух уравнений второй степени. Системы уравнений высших степеней.
§ 4. Иррациональные, показательные и логарифмические уравнения
70. Иррациональные уравнения.
71. Показательные уравнения.
72. Логарифмические уравнения.
73. Разные уравнения. Системы уравнений.
Глава VI. НЕРАВЕНСТВА
74. Свойства неравенств. Действия над неравенствами.
75. Алгебраические неравенства.
§ 2. Решение неравенств
76. Множество решений неравенства. Равносильные неравенства.
77. Графическое решение неравенств.
79. Квадратные неравенства.
80. Неравенства высших степеней. Неравенства, содержащие дробные рациональные функции от х.
81. Иррациональные, показательные и логарифмические неравенства.
82. Неравенства с двумя неизвестными.
Глава VII. ПОСЛЕДОВАТЕЛЬНОСТИ
83. Числовая последовательность.
84. Предел числовой последовательности.
85. Бесконечно малые. Правила предельного перехода.
§ 2. Арифметическая прогрессия
86. Арифметическая прогрессия. Формула общего члена.
87. Свойства арифметической прогрессии.
88. Формула для суммы n членов арифметической прогрессии.
§ 3. Геометрическая прогрессия
89. Геометрическая прогрессия. Формула общего члена.
90. Свойства геометрической прогрессии.
91. Формулы для суммы n членов геометрической прогрессии.
92. Бесконечно убывающая геометрическая прогрессия.
Глава VIII. ТРИГОНОМЕТРИЧЕСКИЕ ФУНКЦИИ УГЛА (ДУГИ)
93. Вектор, проекция вектора.
94. Положительные углы и дуги, меньшие 360°.
95. Углы и дуги, большие 360°.
96. Отрицательные углы. Сложение и вычитание углов.
§ 2. Тригонометрические функции произвольного угла
97. Определение основных тригонометрических функций.
98. Изменение основных тригонометрических функций при изменении угла от 0 до 2pi.
§ 3. Соотношения между тригонометрическими функциями одного и того же угла
99. Основные тригонометрические тождества.
100. Вычисление значений тригонометрических функций по значению одной из них.
101. Значения тригонометрических функций некоторых углов.
§ 4. Четность, нечетность и периодичность тригонометрических функций
102. Четность и нечетность.
103. Понятие периодической функции.
104. Периодичность тригонометрических функций.
§ 5. Формулы приведения
105. Зависимость между тригонометрическими функциями дополнительных углов.
106. Формулы приведения.
Глава IX. ТРИГОНОМЕТРИЧЕСКИЕ ФУНКЦИИ ЧИСЛОВОГО АРГУМЕНТА И ИХ ГРАФИКИ
§ 1. Тригонометрические функции числового аргумента
108. Области определения и области изменения значений тригонометрических функций.
109. Некоторые неравенства и их следствия.
§ 2. Графики тригонометрических функций
110. Первоначальные сведения о таблицах тригонометрических функций.
111. Основные графики.
112. Примеры построения графиков некоторых других тригонометрических функций.
113. Дальнейшие примеры построения графиков функций.
Глава X. ПРЕОБРАЗОВАНИЕ ТРИГОНОМЕТРИЧЕСКИХ ВЫРАЖЕНИЙ
114. Расстояние между двумя точками на плоскости.
115. Косинус суммы и разности двух аргументов.
116. Синус суммы и разности двух аргументов.
117. Тангенс суммы и разности двух аргументов.
118. О формулах сложения для нескольких аргументов.
§ 2. Формулы для двойного и половинного аргумента. Выражение sin na и cos na через степени sin a и cos a
119. Тригонометрические функции двойного аргумента.
120. Выражение sin na и cos na через степени sin a и cos a при натуральном числе n.
121. Тригонометрические функции половинного аргумента.
122. Выражение основных тригонометрических функций аргумента а через tg(a/2).
§ 3. Преобразование в сумму выражений вида sina•cosb, cosa•cosb и sinа•sinb
§ 4. Преобразование в произведение сумм вида
§ 5. Преобразование некоторых выражений в произведения с помощью введения вспомогательного аргумента
127. Преобразование в произведение выражения a•sina + b•cosa.
128. Преобразование в произведение выражений a•sina+b и a•cosa+b
129. Преобразование в произведение выражения a•tga+b.
Глава XI. ОБРАТНЫЕ ТРИГОНОМЕТРИЧЕСКИЕ ФУНКЦИИ И ИХ ГРАФИКИ
130. Функция у = arcsin x (арксинус).
131. Функция y = arccos x (арккосинус).
132. Функция y = arctg x (арктангенс).
133. Функция y = arcctg x (арккотангенс).
134. Пример.
§ 2. Операции над обратными тригонометрическими функциями
135. Тригонометрические операции.
136. Операции сложения (вычитания).
§ 3. Обратные тригонометрические операции над тригонометрическими функциями
137. Функция у = arcsin (sin x).
138. Функция y = arctg (tg x).
Глава XII. ТРИГОНОМЕТРИЧЕСКИЕ УРАВНЕНИЯ И НЕРАВЕНСТВА
139. Уравнение sin х = а.
140. Уравнение cos х = a.
141. Уравнение tg x = a.
142. Уравнение ctg x = a.
143. Некоторые дополнения.
§ 2. Способ приведения к одной функции одного и того же аргумента
145. Некоторые типы уравнений, приводящихся к уравнениям относительно функции одного аргумента.
146. Способ разложения на множители.
147. Решение рациональных тригонометрических уравнений с помощью универсальной тригонометрической подстановки tg(x/2) = t.
§ 3. Некоторые частные приемы решения тригонометрических уравнений и систем
148. Введение вспомогательного аргумента.
149. Преобразование произведения в сумму или разность.
150. Переход к функциям удвоенного аргумента.
151. Решение уравнения типа…
152. Применение подстановок sinx ± соsx = y.
§ 4. Решение тригонометрических неравенств
154. Простейшие тригонометрические неравенства.
155. Примеры тригонометрических неравенств, сводящихся к простейшим.
Часть вторая. ГЕОМЕТРИЯ
156. Точка. Прямая. Луч. Отрезок.
157. Плоскость. Фигуры и тела.
160. Равенство фигур. Движение.
161. Равенство тел.
§ 2. Измерение геометрических величин
162. Сложение отрезков. Длина отрезка.
163. Общая мера двух отрезков.
164. Сравнительная длина отрезков и ломаных.
165. Измерение углов.
166. Радианная мера угла.
167. Измерение площадей.
168. Площадь прямоугольника. Объем прямоугольного параллелепипеда.
Глава XIV. ПЕРПЕНДИКУЛЯРНЫЕ И ПАРАЛЛЕЛЬНЫЕ ПРЯМЫЕ. ЗАДАЧИ НА ПОСТРОЕНИЕ
169. Перпендикуляр и наклонные.
170. Свойство перпендикуляра, проведенного к отрезку в его середине.
171. Параллельные прямые.
172. Углы, образованные двумя параллельными прямыми и секущей.
173. Углы с параллельными или перпендикулярными сторонами.
§ 2. Геометрические места точек. Окружность
174. Геометрическое место точек.
175. Свойство биссектрисы угла.
176. Окружность.
177. Взаимное расположение прямой и окружности. Касательная и секущая.
178. Хорда и диаметр. Сектор и сегмент.
179. Взаимное расположение двух окружностей.
§ 3. Основные задачи на построение
181. Деление отрезка пополам. Построение перпендикуляров.
182. Построение углов.
183. Другие задачи на построение.
Глава XV. ТРЕУГОЛЬНИКИ, ЧЕТЫРЕХУГОЛЬНИКИ
184. Стороны и углы треугольника.
185. Биссектрисы треугольника. Вписанная окружность.
186. Оси симметрии сторон треугольника. Описанная окружность.
187. Медианы и выcоты треугольника.
188. Равенство треугольников.
189. Построение треугольников.
190. Равнобедренные треугольники.
191. Прямоугольные треугольники.
§ 2. Параллелограммы
192. Четырехугольники.
193. Параллелограмм и его свойства.
194. Прямоугольник.
§ 3. Трапеция
196. Трапеция.
197. Средняя линия треугольника.
198. Средняя линия трапеции.
199. Деление отрезка на равные части.
§ 4. Площади треугольников и четырехугольников
200. Площадь параллелограмма.
201. Площадь треугольника.
202. Площадь трапеции.
Глава XVI. ПОДОБИЕ ГЕОМЕТРИЧЕСКИХ ФИГУР
203. Пропорциональные отрезки.
204. Свойства биссектрис внутреннего и внешнего углов треугольника.
§ 2. Подобное преобразование фигур (гомотетия)
205. Определение гомотетичных фигур.
206. Свойства преобразования подобия.
§ 3. Общее подобное соответствие фигур
207. Подобные фигуры.
208. Периметры и площади подобных треугольников.
209. Применение подобия к решению задач на построение.
Глава XVII. МЕТРИЧЕСКИЕ СООТНОШЕНИЯ В ТРЕУГОЛЬНИКЕ И КРУГЕ
210. Углы с вершиной на окружности.
211. Углы с вершиной внутри и вне круга.
212. Угол, под которым виден данный отрезок.
213. Четырехугольники, вписанные в окружность.
214. Пропорциональные отрезки в круге.
215. Задачи на построение.
§ 2. Метрические соотношения в треугольнике
216. Пропорциональные отрезки в прямоугольном треугольнике. Теорема Пифагора.
218. Теорема синусов. Формула Герона.
217. Квадрат стороны, лежащей против острого или тупого утла и треугольнике. Теорема косинусов.
218. Теорема синусов. Формула Герона.
219. Радиусы вписанной и описанной окружностей.
§ 3. Решение треугольников
220. Таблицы функций.
221. Решение треугольников. Сводка основных формул.
222. Решение прямоугольных треугольников.
223. Решение косоугольных треугольников.
Глава XVIII. ПРАВИЛЬНЫЕ МНОГОУГОЛЬНИКИ. ДЛИНА окружности И ПЛОЩАДЬ КРУГА
224. Выпуклые многоугольники.
225. Правильные многоугольники.
226. Соотношения между стороной, радиусом и апофемой.
227. Периметр и площадь правильного n-угольника.
228. Удвоение числа сторон правильного многоугольника.
§ 2. Длина окружности. Площадь круга и его частей
229. Длина окружности.
230. Площадь круга и его частей.
Глава XIX. ПРЯМЫЕ И ПЛОСКОСТИ В ПРОСТРАНСТВЕ
231. Взаимное расположение двух прямых в пространстве.
232. Взаимное расположение прямой линии и плоскости.
233. Взаимное расположение двух плоскостей.
234. Свойства параллельных прямых и плоскостей.
235. Построения в стереометрии.
§ 2. Перпендикулярность прямых и плоскостей
236. Перпендикуляр к плоскости.
237. Перпендикуляр и наклонные.
238. Угол между прямой и плоскостью.
239. Связь между перпендикулярностью и параллельностью прямых и плоскостей.
240. Общий перпендикуляр двух скрещивающихся прямых.
§ 3. Двугранные и многогранные углы
241. Двугранный угол.
242. Взаимно перпендикулярные плоскости.
243. Трехгранные углы.
244. Многогранные углы.
§ 4. Многогранники
245. Многогранники.
246. Правильные многогранники.
Глава XX. МНОГОГРАННИКИ И КРУГЛЫЕ ТЕЛА
247. Цилиндры и призмы.
248. Параллелепипеды.
249. Объемы призм и цилиндров.
250. Площадь боковой поверхности призмы.
251. Площадь поверхности цилиндра.
§ 2. Пирамида. Конус
252. Свойства пирамиды и конуса.
253. Объем пирамиды и конуса.
254. Площадь боковой поверхности правильной пирамиды и конуса.
255. Усеченный конус и усеченная пирамида.
§ 3. Шаровая поверхность. Шар
256. Шар и шаровая поверхность.
257. Объем шара и его частей.
258. Площадь поверхности шара и ее частей.
259. Понятие телесного угла.
Ответы к упражнениям
Приложения

Мэтуэй | Популярные задачи

92) 9(3x) по отношению к x 92+1
1 Найти производную — d/dx бревно натуральное х
2 Оценить интеграл интеграл натурального логарифма x относительно x
3 Найти производную — d/dx
21 Оценить интеграл интеграл от 0 до 1 кубического корня из 1+7x относительно x
22 Найти производную — d/dx грех(2x)
23 Найти производную — d/dx
41 Оценить интеграл интеграл от cos(2x) относительно x
42 Найти производную — d/dx 1/(корень квадратный из х)
43 Оценка интеграла 9бесконечность
45 Найти производную — d/dx х/2
46 Найти производную — d/dx -cos(x)
47 Найти производную — d/dx грех(3x)
68 Оценить интеграл интеграл от sin(x) по x
69 Найти производную — d/dx угловой синус(х)
70 Оценить предел ограничение, когда x приближается к 0 из (sin(x))/x 92 по отношению к х
85 Найти производную — d/dx лог х
86 Найти производную — d/dx арктан(х)
87 Найти производную — d/dx бревно натуральное 5х92

Мэтуэй | Популярные задачи

92
1 Найти точное значение грех(30)
2 Найти точное значение грех(45)
3 Найти точное значение грех(30 градусов)
4 Найти точное значение грех(60 градусов)
5 Найти точное значение загар (30 градусов)
6 Найти точное значение угловой синус (-1)
7 Найти точное значение грех(пи/6)
8 Найти точное значение cos(pi/4)
9 Найти точное значение грех(45 градусов)
10 Найти точное значение грех(пи/3)
11 Найти точное значение арктан(-1)
12 Найти точное значение cos(45 градусов)
13 Найти точное значение cos(30 градусов)
14 Найти точное значение желтовато-коричневый(60)
15 Найти точное значение csc(45 градусов)
16 Найти точное значение загар (60 градусов)
17 Найти точное значение сек(30 градусов)
18 Найти точное значение cos(60 градусов)
19 Найти точное значение соз(150)
20 Найти точное значение грех(60)
21 Найти точное значение cos(pi/2)
22 Найти точное значение загар (45 градусов)
23 Найти точное значение arctan(- квадратный корень из 3)
24 Найти точное значение csc(60 градусов)
25 Найти точное значение сек(45 градусов)
26 Найти точное значение csc(30 градусов)
27 Найти точное значение грех(0)
28 Найти точное значение грех(120)
29 Найти точное значение соз(90)
30 Преобразовать из радианов в градусы пи/3
31 Найти точное значение желтовато-коричневый(30)
35 Преобразовать из радианов в градусы пи/6
36 Найти точное значение детская кроватка(30 градусов)
37 Найти точное значение арккос(-1)
38 Найти точное значение арктический(0)
39 Найти точное значение детская кроватка(60 градусов)
40 Преобразование градусов в радианы 30
41 Преобразовать из радианов в градусы (2 шт.

Сумма геометрической прогрессии конечной: Сумма членов конечной геометрической прогрессии, первый член которой равен 1, а знаменатель положителен, равна 21/16,…

111. Геометрическая прогрессия

Геометрическая прогрессия – это числовая последовательность, первый член которой отличен от нуля, а каждый член, начиная со второго, равен предыдущему, умноженному на одно и то же отличное от нуля число , где – знаменатель прогрессии: ().

Общий вид геометрической прогрессии:

:: ; ; ; …; ; …

Геометрическая прогрессия является возрастающей при и убывающей при .

Например, :: 2; 6; 18; 54; …; – возрастающая прогрессия; :: 250; 50; 10; …; – убывающая прогрессия.

Если заданы первый член и знаменатель , то -й член геометрической прогрессии определяют по формуле:

.

Сумму первых членов Геометрической прогрессии находят по формуле: .

Свойства геометрической прогрессии.

1. Квадрат каждого среднего члена прогрессии равен произведению равноотстоящих от него членов:

; ().

2. В конечной геометрической прогрессии произведения двух членов, равноотстоящих от ее концов, равны между собой и равны произведению крайних членов:

:: ; ; ; . ..; ; ; …; ; ;

.

Пример 7. Найдите первый и последний члены геометрической прогрессии, которая состоит из четырех членов, если и .

Решение. Подставим исходные данные в формулу:

.

Найдем по формуле и получим: .

Ответ. и .

Пример 8. В геометрической прогрессии (): . Найдите сумму восьми первых членов прогрессии .

Решение. ; ; , тогда запишем исходную систему так: .

Разделим почленно второе уравнение на первое. Получим:

.

Найдем из первого уравнения системы: .

По формуле для суммы найдем: .

Ответ. .

Пример 9. Шесть чисел составляют геометрическую прогрессию. Сумма первых трех чисел равна 168, а сумма последних трех чисел равна 21. Найдите эти числа.

Решение. Из условия задачи составим систему уравнений:

Найдем , для этого разделим первое уравнение на второе: .

Найдем из первого уравнения: .

Ответ. 96; 48; 24; 12; 6; 3.

Пример 10. Найдите сумму , .

Решение. По условию задания можно сделать вывод о том, что: – это геометрическая прогрессия. Найдем первый член прогрессии, знаменатель и общее количество ее членов: ; ; . Тогда .

Ответ. .

Бесконечно убывающая геометрическая прогрессия – это такая бесконечная геометрическая прогрессия (), у которой знаменатель .

Сумму бесконечно убывающей геометрической прогрессии находят по формуле:

.

Пример 11. Запишите периодическую дробь 0,4545…=0,(45) как обыкновенную.

Решение. Запишем периодическую дробь в виде бесконечной суммы обыкновенных дробей: .

Слагаемые представляют собой бесконечно убывающую геометрическую прогрессию со знаменателем и первым членом , а полученная сумма – это сумма этой прогрессии.

Используя формулу суммы бесконечно убывающей геометрической прогрессии, получим: .

Ответ. .

Пример 12. Найдите бесконечно убывающую геометрическую прогрессию, если , а сумма .

Решение. Используя формулу суммы бесконечно убывающей геометрической прогрессии найдем:

.

Ответ. :: ; ; ; …

Пример 13. Найдите бесконечно убывающую геометрическую прогрессию, если ее сумма равна , а сумма ее первых четырех членов равна .

Решение. Из условия задачи запишем систему:

.

Подставим правую часть первого уравнения во второе уравнение:

И .

Тогда найдем два значения :

1) ; 2) .

Ответ. 1) :: ; ; ; ; …; 2) :: ; ; ; ; … .

Пример 14. Сумма бесконечно убывающей геометрической прогрессии , а сумма квадратов всех ее членов . Найти четвертый член прогрессии.

Решение. Найдем знаменатель прогрессии, которая состоит из квадратов членов: ; ; ; …; ; … : .

Тогда составим систему уравнений: .

Возведем первое уравнение в квадрат: .

Разделим второе уравнение системы на первое:

Тогда ; .

Ответ. .

< Предыдущая   Следующая >

Сумма первых N членов конечной геометрической прогрессии Калькулятор

✖Первый член конечной GP — это значение, соответствующее первому члену конечной геометрической прогрессии.ⓘ Первый срок конечного GP [a]

+10%

-10%

✖Общее отношение конечной GP — это отношение любого члена к предыдущему члену геометрической прогрессии, в которой не существует бесконечной суммы членов.ⓘ Общий коэффициент конечного GP [r]

+10%

-10%

✖Номер индекса n-го члена конечной ГП — это значение n для n-го члена или положение n-го члена в конечной геометрической прогрессии.ⓘ Номер индекса n-го члена конечной ГП [n]

+10%

-10%

✖Сумма первых N членов конечной ГП — это сумма членов, начиная с первого по n-й член данной конечной геометрической прогрессии. ⓘ Сумма первых N членов конечной геометрической прогрессии [Sn]

⎘ копия

👎

Формула

сбросить

👍

Сумма первых N членов конечной геометрической прогрессии Решение

ШАГ 0: Сводка предварительного расчета

ШАГ 1. Преобразование входов в базовый блок

Первый срок конечного GP: 3 —> Конверсия не требуется
Общий коэффициент конечного GP: 2 —> Конверсия не требуется
Номер индекса n-го члена конечной ГП: 4 —> Конверсия не требуется

ШАГ 2: Оцените формулу

ШАГ 3: Преобразуйте результат в единицу вывода

45 —> Конверсия не требуется

< 2 Конечная геометрическая прогрессия Калькуляторы

Сумма первых N членов конечной геометрической прогрессии формула

Сумма первых N членов конечной ЗП = (Первый срок конечного GP*((Общий коэффициент конечного GP^Номер индекса n-го члена конечной ГП)-1))/(Общий коэффициент конечного GP-1)
Sn = (a*((r^n)-1))/(r-1)

Что такое геометрическая прогрессия?

В математике геометрическая прогрессия или просто GP, также известная как геометрическая последовательность, представляет собой последовательность чисел, в которой каждый член после первого находится путем умножения предыдущего на фиксированное действительное число, называемое обыкновенным отношением. Например, последовательность 2, 6, 18, 54,… является геометрической прогрессией со знаменателем 3. Если сумма всех членов прогрессии является конечным числом или если существует бесконечная сумма прогрессии, то мы скажем, это бесконечная геометрическая прогрессия или бесконечная GP. А если бесконечной суммы прогрессии не существует, то это конечная геометрическая прогрессия или конечная ВП. Если абсолютное значение общего отношения больше 1, то ЗП будет Конечным ЗП, а если оно меньше 1, то ЗП будет Бесконечным ЗП.

Share

Copied!

геометрия — Визуальная интуиция для суммы КОНЕЧНОГО геометрического ряда

спросил

Изменено 9 месяцев назад

Просмотрено 1к раз

$\begingroup$

Меня интересуют интуитивные визуальные объяснения суммы конечного геометрического ряда.

Я знаю, что есть несколько довольно «интуитивных» объяснений (в том числе и на этом сайте) , но я не видел ни одного визуального интуитивного понимания.

Если кто-нибудь знает о них и поделится ими, я был бы очень признателен!

Спасибо!


Конечный геометрический ряд . Все ответы до сих пор относились к бесконечному случаю.

Спасибо!

  • последовательности и серии
  • геометрия
  • суммирование
  • мягкий вопрос
  • интуиция
$\endgroup$

3

$\begingroup$

Вот еще одно доказательство (скопировано с блестящего.org) бесконечного ряда, но для произвольного $r<1$. Интересно, можно ли его адаптировать для конечного случая, если подумать о такой трапеции вместо треугольника...

$\endgroup$

3

$\begingroup$

Если кто-то выживет, ограничиваясь только натуральными числами, предлагаю иллюстрацию с помощью деревьев.

Каждому из $n$ членов геометрического ряда со знаменателем $r$ соответствует уровень в (совершенном) $r$-арном дереве $T_n$. Сумма $s$ определяется количеством узлов в $T_n$.

Основная идея приходит из сравнения $T_n$ с $T_{n+1}$, т.е. сравнения ряда из $n$ и $n+1$ терминов: 9{n+1}-1}{r-1}$

Итого , рассматривая следующий член ряда, вы увеличиваете свою сумму на известную величину (шаг 2), что соответствует умножению оригинала на известную величину (шаг 3).

$\endgroup$

$\begingroup$

Смотрите эти изображения:

Это графическое объяснение суммы геометрической прогрессии отношения $\frac{1}{2}$.

$\endgroup$

2

$\begingroup$

Нарисуйте отрезок длиной в одну единицу.

Галочка в первой трети слева.

С правой стороны отметьте первую треть слева.

С правой стороны отметьте первую треть слева.

С правой стороны отметьте первую треть слева.

Когда вы закончите, у вас будет бесконечная сумма для $a=\frac13,r=\frac23$. 94$$.

Теперь представьте себе прямоугольник со сторонами 1 и 2 . Теперь возьмите еще два таких прямоугольника и поставьте их на соседние позиции, они образуют большой прямоугольник со сторонами 2 и 2 . Теперь снова возьмите два больших прямоугольника, чтобы повторить процесс до 4 раз. (См. схему). Теперь все, что вам нужно, это найти общую площадь этих прямоугольников. Здесь у двух последовательных прямоугольников одна сторона общая, а площадь в два раза больше.

Теперь давайте сделаем еще один шаг и нарисуем следующий прямоугольник, площадь которого будет в два раза больше предыдущего. Теперь мы хотим проверить, больше или меньше эта площадь, чем S. 95-2=32-2=30}$$

Когда вы думаете об обобщении этого метода, нам нужно на $\color{red}{k}$ больше предыдущих прямоугольников. Или мы должны разделить эту область на один шаг дальше на k частей, и мы будем использовать 1 часть, чтобы снова скрыть, поэтому у нас есть k-1 частей для разделения, и в конечном итоге мы останемся с наименьшим прямоугольником. Значит, площадь равна k-1 раз.

$$ S(k-1)=A_{n+1}-A_1$$

$\endgroup$

$\begingroup$

Метод треугольника/трапеции, вероятно, является наиболее традиционным, однако существует еще один метод, основанный на его упрощении. 9n$$

Я полагаю, что кто-то уже придумал этот точный метод, однако я не смог найти аналогичный метод нигде в Интернете.

$\endgroup$

7.4.1: Суммы конечного геометрического ряда

  1. Последнее обновление
  2. Сохранить как PDF
  • Идентификатор страницы
    14795
  • Нахождение суммы конечного геометрического ряда

    Вы копите деньги на летний лагерь. Вы вносите 100 долларов первого числа каждого месяца на свой сберегательный счет. Счет растет со скоростью 0,5% в месяц. Сколько денег на вашем счету в первый день 9 числа месяца?


    Сумма конечного геометрического ряда

    Мы обсудили, как использовать калькулятор для нахождения суммы любого ряда при условии, что мы знаем n 9{п-1}\).

    Наконец, решим следующую задачу.

    Первого числа каждого года Чарли вносит 1000 долларов на свой инвестиционный счет. Счет растет со скоростью 8% в год. Сколько денег на счету в первый день 11 -го года.

    Сначала рассмотрим, что здесь происходит в первый день каждого года. В первый день первого года вносится 1000 долларов. В первый день второго года вносится 1000 долларов, а ранее внесенные 1000 долларов приносят 8% годовых или увеличиваются в 1,08 раза (108%). В первый день третьего года вносится еще 1000 долларов, депозит предыдущего года приносит 8% годовых, а первоначальный депозит приносит 8% годовых на два года (мы умножаем на 1,08 9). 0200 2 ):

    Сумма за год 1: 1000

    Сумма за год 2: 1000 + 1000(1,08)

    Сумма за год 3: 1000 + 1000(1,08) + 1000(1,08) 9 0200 2

    Сумма Год 4: 1000 + 1000(1,08) + 1000(1,08) 2 + 1000(1,08) 3

    \(\ \четверка\четверка\четверка\четверка\)⋮

    Сумма Год 11: 1 000 + 1000 (1,08) + 1000(1,08) 2 + 1000(1,08) 3 + … + 1000(1,08) 9 + 1000(1,08) 10

    ∗ В этом ряду 11 слагаемых, потому что в первый день 11 9{11}\right)}{1-1.08}=16645,48746 \приблизительно \$ 16 645,49\)


    Примеры

    Пример 1

    9 месяц.

    Решение

    В этой серии 9 терминов, потому что в первый день 9 -го месяца вы вносите свой последний депозит, а первоначальный депозит приносит проценты в течение 8 месяцев.

    Эта серия геометрическая. Первый член равен 100, обыкновенное отношение равно 1,005 и n = 9.{n-1}\)

    Пример 4

    Первого числа каждого месяца Сэм вносит 50 долларов на счет, который ежемесячно приносит 0,5% годовых. {48},\\
    \quad \uparrow \quad\quad\quad\quad\quad\quad\quad\quad\quad\quad\quad\quad\quad\quad\quad\quad\quad\quad\quad\quad\quad\quad\ quad\quad\quad\quad\quad\uparrow\\
    \text {последний депозит} \quad\quad\quad\quad\quad\quad\quad\quad\quad\quad\quad\quad\quad\quad\quad \quad\quad\quad\quad \text { первый депозит }
    \end{aligned}\)

    Обратите внимание, что на первый депозит начисляются проценты в течение 48 месяцев, а на последний депозит проценты не начисляются. Теперь мы можем найти сумму, используя \(\ a_{1}=50\), \(\ r=1,005\) и \(\ n=49{7} a_{n}=-\frac{63}{2}\) и \(\ r=-\frac{1}{2}\)

    Решите следующие текстовые задачи, используя формулу суммы геометрического ряда.

    1. Бабушка и дедушка Сапны вносят 1200 долларов на сберегательный счет колледжа в день ее -го дня рождения 5  года. Они продолжают вносить этот депозит на день рождения каждый год, пока не сделают последний депозит в день ее рождения 18   года. Если счет приносит 5% годовых, сколько остается после окончательного депозита?
    2. Джереми хочет накопить 10 000 долларов за пять лет. Если он вносит ежегодные депозиты первого числа каждого года и на счете зарабатывается 4,5% годовых, сколько он должен вносить каждый год, чтобы иметь 10 000 долларов на счете после окончательного депозита первого из 6 год. Округлите ответ до ближайших 100 долларов.

    Ответы на проблемы с обзором

    Чтобы просмотреть ответы на обзор, откройте этот PDF-файл и найдите раздел 11.10.


    Словарь

    Срок Определение
    индукция Индукция — это метод математического доказательства, обычно используемый для установления того, что данное утверждение верно для всех положительных целых чисел.
    серия Серия — это сумма членов последовательности.

    Модуль числа с x: Уравнения с модулями. Модули

    1.2 Решение линейных уравнений с модулем

    Для начала, стоит вспомнить, что такое модуль числа. Итак, абсолютной величиной или модулем числа называется само число х, если х положителен, число (-х), если х отрицателен, или нуль, если х=0. Значение модуля может быть только положительным.

    Чтобы понять решение параметрических уравнений, содержащих знак модуля, лучше всего продемонстрировать решение наглядно, т.е. привести примеры:

    Пример 1. Решить уравнение |x-2|=b.

    Так как, по определению модуля, |x-2| , то при b<0 данное уравнение решений не имеет. Если b=0, то уравнение имеет решение х=2.

    Если b>0, то решениями уравнения являются числа x=2+b и x=2-b.

    Ответ: при b<0 решений нет, при b=0 х=2, при b>0 х=2+b и x=2-b.

    Пример 2. Решить уравнение |x-a|=|x-4|. Удобнее всего данное уравнение решить методом интервалов, для двух случаев:

    a ;

    4 .

    1. Первый интервал:

        ;

    Второй интервал:

        , т. е. если а<4, то  .

    Третий интервал:

     а=4, т.е. если а=4, то  .

    2. Первый интервал:

     а=4,  .

    В торой интервал:

         a>4,т.е. если 4<а, то 

    Третий интервал:

       

    Ответ: при а=4 х-любое;, при а<4  .

    Пример 3. Для каждого значения параметра а найти все значения х, удовлетворяющие уравнению |x+3|– a| x – 1| =4.

    Рассмотрим 3 промежутка: 1)  , 2)  , 3)   и решим исходное уравнение на каждом промежутке.

    1.  ,  .

    При а=1 уравнение не имеет решений, но при а 1 уравнение имеет корень  . Теперь надо выяснить, при каких а х попадает на промежуток x< – 3, т.е.  ,  ,  ,  . Следовательно, исходное уравнение на x< – 3 имеет один корень   при  , а на остальных а корней не имеет.

    2.  .  .

    При а= – 1 решением уравнения является любое х; но мы решаем на промежутке  . Если а 1, то уравнение имеет один корень х=1.

    3.  .  .

    При а=1 решением является любое число, но мы решаем на  . Если а 1, то х=1.

    Ответ: при    ; при а= – 1   и при а 1 х=1; при а=1   и при а 1 х=1.

    1.3 Решение квадратных уравнений с параметром

    Для начала напомню, что квадратное уравнение – это уравнение вида  , где а, b и с – числа, причем, а 0.

    Условия параметрических квадратных уравнений могут быть различны, но для решений всех их нужно применять свойства обыкновенного квадратного уравнения  :

    а) Если D>0, а>0, то уравнение имеет два действительных различных корня, знаки которых при с>0 одинаковые и противоположны по знаку коэффициента b, а при с<0, причем по абсолютной величине больше тот, знак которого противоположен коэффициенту b.

    б) Если D=0, а>0, то уравнение имеет два действительных и равных между собой корня, знак которых противоположен знаку коэффициента b.

    в) Если D<0, а>0, то уравнение не имеет действительных корней.

    Аналогично можно представить свойства корней при а<0. Кроме того, в квадратных уравнениях справедливы следующие утверждения:

    Если поменять местами коэффициенты а и

    с, то корни полученного квадратного уравнения будут обратны корням данного.

    Если поменять знак коэффициента b, корни полученного квадратного уравнения будут противоположны корням данного.

    Если коэффициенты а и с разных знаков, то уравнение имеет действительные корни.

    Пример1. Найти все значения параметра а, для которых квадратное уравнение  : а) имеет два различных корня; б) не имеет корней; в) имеет два равных корня.

    Данное уравнение по условию является квадратным, поэтому а -1. Рассмотрим дискриминант данного уравнения:

    При а>-1 уравнение имеет два различных корня, т.к. D>0, при a<-1 уравнение корней не имеет, т.к. D<0, а двух одинаковых корней это уравнение иметь не может, т.к. D=0 при а=-1, а это противоречит условию задачи.

    Пример2. Решить уравнение 

    При а=0 уравнение является линейным 2х+1=0, которое имеет единственное решение х=-0.5. А при а 0, уравнение является квадратным и его дискриминант D=4-4a.

    При а>1 D<0 поэтому уравнение корней не имеет. При а=1 D=0, поэтому уравнение имеет два совпадающих корня  =-1.

    При a<1, но а 0, D>0 и данное уравнение имеет два различных корня

     ;  .

    Ответ:   и   при a<1, но а 0; х=-0.5 при а=0;  =-1 при а=1.

    Пример3. Корни уравнения   таковы, что  . Найдите а.

    По теореме Виета   и  . Возведём обе части первого равенства в квадрат:  . Учитывая, что , а  , получаем:   или  ,    . Проверка показывает, что все значения   удовлетворяют условию.

    Ответ: 

    Модуль

    Модулем положительного числа называют само это число; модулем отрицательного числа называют число, ему противоположное; модуль нуля равен нулю.

    \(|a|=\begin{cases} a, \;\; если \;  a>0 \\ 0, \; если\;\; a=0\\ -a,\; если \;\;  a<0  \end{cases}\)

    Второе название модуля – «абсолютное значение действительного числа».

    Фактически модуль делает всё, что находится внутри него положительным. Поэтому чтобы правильно его раскрыть, необходимо сначала выяснить знак выражения внутри него:

    — если подмодульное выражение положительно, модуль просто убирается. 4+1\)

    Пример. Вычислить значение выражения \(|7-x|-|x+3|\), при \(x>12\).

    Решение: При любом \(x\) большем \(12\), первое подмодульное выражение будет отрицательно, а второе – положительно. Соответственно, первый модуль будет раскрываться с минусом, а второй – с плюсом (значит перед ним останется минус, который стоял перед ним до раскрытия):

    \(|7-x|-|x+3|=-(7-x)-(x+3)=-7+x-x-3=-10\)

    Ответ: \(-10\)



    Геометрическое определение модуля

    \(|a|\) — это расстояние от \(0\) до числа \(a\) на числовой оси

    Пример. Чему равен \(|5|\)  и \(|-5|\)?

    Представим числовую ось и отметим на ней точки \(5\) и \(-5\). Какое будет расстояние от нуля до этих точек? Очевидно \(5\).


    Значит ответ: \(|5|=5\),   \(|-5|=5\).

    Так как модуль это расстояние, а расстояние не может выражаться отрицательным числом, то он всегда положителен.

    Понимать легче второе определение, но практике удобнее использовать первое.

    Решение простейших уравнений с модулем

    Уравнения вида \(|f|=g\) решается с помощью перехода к совокупности   \( \left[ \begin{gathered}f= g\\  f=-g\end{gathered}\right.\) , при условии, что \(g≥0\).

    Сначала об условии \(g≥0\). Откуда оно берется? Из определения модуля, ведь модуль всегда неотрицателен (то есть, положителен или равен нулю). Поэтому условие \(g≥0\) обязательно. Иначе уравнение не будет иметь решения.

    Теперь о совокупности. Почему уравнение распадается на два? Давайте, к примеру, рассмотрим уравнение \(|x|=3\). Какое число под модулем будет равно \(3\)? Конечно \(3\) и \(-3\), потому что \(|3|=3\), \(|-3|=3\). Корни уравнения \(|x|=3\): \(3\) и \(-3\). Логично? Логично! В общем виде получается, что подмодульное выражение \(f\) должно быть равно \(g\) и \(-g\). Иначе равенство не получится.

    Пример.   Решить уравнение:

    \(|x-1|=3x\)

    Найдем ограничения уравнения. Запишем его немного правее от основного решения

                                   \(3x≥0\)
                                   \(x≥0\)

     

    Когда ограничение записано —  можно со спокойной душой решать уравнение. Избавимся от модуля и перейдем к совокупности уравнений

    \( \left[ \begin{gathered}x-1=3x\\ x-1=-3x\end{gathered}\right.\)

     

    Перед нами 2 линейных уравнения. Решаем их с помощью известного заклинания: «иксы влево, числа вправо»

    \( \left[ \begin{gathered}x-3x=1\\ x+3x=1\end{gathered}\right. \)

     

    Приведем подобные слагаемые

    \( \left[ \begin{gathered}-2x=1\\ 4x=1\end{gathered}\right.\)


    Поделим первое уравнение на \(-2\), второе на \(4\).

    \( \left[ \begin{gathered} x=-\frac{1}{2}\\ x=\frac{1}{4}\end{gathered}\right.\)


    Корень \(-\)\(\frac{1}{2}\) – не подходит, т.к. \(x≥0\). Остается корень \(\frac{1}{4}\), его и запишем в ответ

    Ответ:  \(\frac{1}{4}\)

    Решение простейших неравенств с модулем

    Неравенство вида \(|f|< c\) решается с помощью перехода к двойному неравенству   \( -c< f< c\) , при условии, что \(c>0\).

    Начнем опять с условия. Почему \(c>0\)? Потому что, иначе неравенство не будет иметь решения. Здесь все также как в уравнениях. В самом деле, когда, например, модуль икса меньше \(-7\)? Никогда!

    Теперь разберем неравенство \(|x|<3\). Какие иксы нам подойдут? Все от \(-3\) до \(3\). Иначе говоря, икс должен лежать между \(-3\) и \(3\). Это утверждение можно записать вот так \(-3< x <3\) либо системой \(\begin{cases}x<3\\x > -3\end{cases}\). В любом случае ответ будет \(xϵ (-3;3)\).

    Неравенство вида \(|f|>c\) решается с помощью перехода к совокупности неравенств \( \left[ \begin{gathered} f>c\\ f< -c\end{gathered}\right.\), при условии, что \(c≥0\).

    А здесь почему \(c≥0\)? Потому что иначе решать нечего: если \(c\) отрицательно, то модуль абсолютно любого икса нам подойдет. И значит ответ, икс – любое число.

    Теперь о переходе. Рассмотрим неравенство \(|x|>3\). Какие иксы нам подойдут? Все, модуль которых больше трех, то есть от минус бесконечности до \(-3\) и от \(3\) до плюс бесконечности. Записывая системой получим \(\begin{cases}x>3\\x < -3\end{cases}\). Ответ будет \(x ϵ (-∞;-3)⋃(3;∞)\).

    \(|3x-7|≤8\)

    \(|3x-11|≥11\)

    \(-8≤3x-7≤8\) \(|+7\)

    \( \left[ \begin{gathered}3x-11≥11\\ 3x-11≤-11\end{gathered}\right.\)

    \(-1≤3x≤15\)

     

    \( \left[ \begin{gathered}3x≥22\\ 3x≤0\end{gathered}\right.\)

    \(-\frac{1}{3}≤x≤5\)

     

    \( \left[ \begin{gathered}x≥\frac{22}{3}\\ x≤0\end{gathered}\right. \)

    Ответ: \([ -\frac{1}{3};5]\)

     

    Ответ: \( (-\infty;0]\cup [ \frac{22}{3};\infty)\)

    Смотрите также:
    Свойства модуля

    Теория чисел. Модульная арифметика

    ◀ Теория чисел. Алгоритм Евклида ▶

    Содержание

    Пусть \(n\) — положительное целое число. Обозначим множество \([0..n-1]\) через \(\mathbb{Z}_n\).

    Мы считаем два целых числа \(x, y\) одинаковыми, если \(x\) и \(y\) отличаются на a кратно \(n\), и мы записываем это как \(x = y \pmod{n}\) и говорим, что \(x\) и \(y\) конгруэнтны по модулю \(n\). Мы можем опустить \(\pmod{n}\), когда ясно из контекста. Каждое целое число \(x\) конгруэнтно некоторому \(y\) в \(\mathbb{Z}_n\). Когда мы добавляем или вычесть кратные \(n\) из целого числа \(x\), чтобы получить некоторое \(у\в\mathbb{Z}_n\), мы говорим уменьшает \(x\) по модулю \(n\), а \(y\) является остатком .

    Мы могли бы выбрать разные наборы для \(\mathbb{Z}_n\), например мы могли бы добавить \(n\) к каждому элементу, но по умолчанию будет \([0..n-1]\). Элементы в этом конкретном представлении \(\mathbb{Z}_n\) называются наименьшими остатками .

    Пример : \(38 = 3 \pmod{5}\), так как \(38 = 7\умножить на 5 + 3\). \(-3 = 11 \pmod{14}\), так как \(-3 = (-1)\умножить на 14 + 11\).

    Каков наиболее естественный способ выполнения арифметических действий в \(\mathbb{Z}_n\)? Учитывая два элемента \(x, y \in \mathbb{Z}_n\), мы можем складывать, вычитать или умножать их как целые числа, а затем результат будет соответствовать одному из элементов в \(\mathbb{Z}_n\).

    Пример : \(6 + 7 = 1 \pmod{12}\), \(3 \times 20 = 10 \pmod{50}\), \(12 — 14 = 16 \pmod{18}\).

    Эти операции ведут себя так же, как их обычные аналоги. Однако понятия размера нет. Говоря \(0

    Division

    Division явно отсутствует в приведенном выше обсуждении. Если \(y\) делит \(x\) как целые числа, то можно предположить, что мы могли бы использовать обычное определение. Посмотрим, к чему это приведет: у нас есть \(10 = 4 \pmod{6}\). Разделение обоих сторон на \(2\) дает неверное уравнение \(5 = 2 \pmod{6}\).

    Таким образом, мы должны изменить значение деления. Интуитивно деление должно «отменять умножение», т. е. разделить \(x\) на \(y\) означает найти число \(z\) такое, что \(y\) умножить на \(z\) равно \(x\). Проблема выше в том, что существуют разные кандидаты на \(z\): в \(\mathbb{Z}_6\) и 5, и 2 дают 4 при умножении на 2.

    Какой ответ мы должны выбрать для «\(4 / 2\)», \( 5\) или \(2\)? Мы могли бы ввести некоторые произвольные соглашения, такие как выбор наименьшего ответ при рассмотрении наименьшего остатка как целого числа, но тогда деление будет вести себя странно.

    Вместо этого мы требуем уникальности, то есть \(x\), деленное на \(y\) по модулю \(n\), составляет всего определяется, когда существует уникальный \(z \in \mathbb{Z}_n\) такой, что \(x = y z\).

    Мы можем получить условие на \(у\) следующим образом. Предположим, \(z_1 y = z_2 y \pmod {n}\). Тогда по определению это означает для некоторого \(k\) имеем \(y(z_1 — z_2) = k n\). Пусть \(d\) будет наибольшим общий делитель \(n\) и \(y\). Тогда \(n/d\) делит \(z_1 — z_2\) поскольку он не может делить \(y\), мы имеем

    \[ z_1 y = z_2 y \pmod {n} \]

    тогда и только тогда, когда

    \[ z_1 = z_2 \pmod {n/d} . \]

    Таким образом, уникальный \(z\) существует по модулю \(n\), только если наибольшее общее делитель \(y\) и \(n\) равен 1.

    Обратные

    Мы увидим, что существует единственное \(z\) тогда и только тогда, когда можно найти \(w \in \mathbb{Z}_n\) такое, что \(y w = 1 \pmod {n}\). Если такое \(w\) существует, то оно должно быть уникальным: предположим, что \(y w’\) также равно 1. Тогда, умножая обе части \(y w = y w’\) через \(w\) дает \(w y w = w y w’\), откуда следует \(w = w’\) так как \(wy = 1\). Когда он существует, мы называем это уникальное \(w\) 9{-1}\) существует, и если да, то как его найти? Поскольку в \(\mathbb{Z}_n\) всего \(n\) элементов, мы можем умножить каждый элемент по очереди на \(y\) и посмотреть, получим ли мы 1. {-1}\), если существует инверсия \(y\), иначе ответ не определено. 9{-1}) = 2 \pmod{6}\).

    ◀ Теория чиселАлгоритм Евклида ▶

    Содержание


    Бен Линн [email protected] 💡

    Математическая задача: Модуль — вопрос № 4325, алгебра, уравнение

    Найдите x в уравнении по модулю:

    47x = 4 (mod 9)

    Подсказка — прочитайте, какое число 47x разделить на 9 (по модулю 9) дает остаток 4.

    Правильный ответ:

    x =  2

    Пошаговое объяснение:

    47x = 9к + 4; k,x−целое число k=(47x−4)/9k1=(47⋅1−4)/9=943=497=4,7778k2=(47⋅2−4)/9=10k3​ =(47⋅ 3−4)/9=9137​=1592​≐15,2222 k4​=(47⋅ 4−4)/9=9184​=2094​≐20,4444 …. x=2


    Нашли ошибку или неточность? Не стесняйтесь

    пишите нам

    . Спасибо!

    Советы по связанным онлайн-калькуляторам

    Вы решаете задачи Диофанта и ищете калькулятор целочисленных уравнений Диофанта?
    У вас есть линейное уравнение или система уравнений и вы ищете ее решение? Или у вас есть квадратное уравнение?

    Чтобы решить эту математическую задачу, вам необходимо знать следующие знания:

    • алгебра
    • уравнение
    • целочисленное уравнение
    • делимость
    • 90 108 основные функции
    • по модулю
    • числа
    • целые числа
    • натуральные числа
    Уровень задачи:
    • практика для 14-летних
    • старшая школа
    • Большое число
      Какой остаток при делении 10 на 9 до 47 — 111?
    • Напоминание и частное
      Даны числа A = 135, B = 315. Найдите наименьшее натуральное число R, большее единицы, так, чтобы отношения R:A, R:B были с остатком 1.
    • Делимость
      Является ли число 761082 точно делится на 9? (результат — целое число и/или остаток равен нулю)
    • Остаток 33031
      Найдите число, которое при делении на 28 дает соотношение 606 и остаток 23.
    • Остаток 34441
      Найдите остаток после деления суммы на 1! +2! +3! +. … . +300! число 13.
    • Неизвестное целое число
      Найдите наименьшее целое число: При делении на 2 остаток равен 1. При делении на 3. Остаток равен 2. При делении на 4. Остаток равен 3. При делении на восемь остаток равен 7. При делении на 9 остаток равен 8.
    • Трехзначное 8002
      Найдите наибольшее трехзначное число, которое дает остаток 1 при делении на три, дает остаток 2 при делении на четыре, дает остаток 3 при делении на пять, и дает остаток 4 при делении на шесть.
    • Делимый 9331
      Число X — наименьшее натуральное число, половина которого делится на три, треть — на четыре, четверть — на одиннадцать, а его половина дает остаток 5 при делении на семь. Найдите это число.
    • Несколько систем счисления
      Найдите значение x, для которого 312 четыре +52 x = 96 десять . Подсказка: четыре, х и десять — это основание заданного числа.
    • Остаток 5594
      Какое число мы разделили на 55, если отношение равно 90,16, а остаток 0,04?
    • Отец 2
      Отец отдал 1/3 своей земли дочери, а оставшуюся часть сыну. Сын жертвует 3/4 своей земли под детскую площадку. На половине оставшейся площади он построил дом, а другую половину продал за 1 миллион рупий. 1) давать сыновьям часть в виде дроби
    • Остатки
      Дан набор чисел { 170; 244; 299; 333; 351; 391; 423; 644}. Разделите эти числа на число 66 и определите множество остатков. В результате запишите сумму этих остатков.
    • Дата игры
      Пусть сейчас вторник. Какой день наступает через 229 дней? Запишите результат в виде числа: 1=понедельник, 2=вторник, 3=среда, 4=четверг, 5=пятница, 6=суббота, 7=воскресенье.
    • Рынок открыт
      Определенный рынок открывается для продаж каждый 7-й день недели.

    Координаты середины отрезка формула в пространстве: Середина отрезка. Координаты середины отрезка

    Математика: Справ. материалы

    Математика: Справ. материалы
      

    Гусев В. А., Мордкович А. Г. Математика: Справ. материалы: Кн. для учащихся.— М.: Просвещение, 1988.— 416 с.

    В книге дано краткое изложение основных разделов школьных курсов алгебры и начал анализа, геометрии. Книга окажет помощь в систематизации и обобщении знаний по математике.



    Оглавление

    СЛОВО К УЧАЩИМСЯ
    ГЛАВА I. ЧИСЛА
    § 1. Натуральные числа
    2. Арифметические действия над натуральными числами.
    3. Деление с остатком.
    4. Признаки делимости.
    5. Разложение натурального числа на простые множители.
    6. Наибольший общий делитель нескольких натуральных чисел.
    7. Наименьшее общее кратное нескольких натуральных чисел.
    8. Употребление букв в алгебре. Переменные.
    § 2. Рациональные числа
    10. Равенство дробей. Основное свойство дроби. Сокращение дробей.
    11. Приведение дробей к общему знаменателю.
    12. Арифметические действия над обыкновенными дробями.
    13. Десятичные дроби.
    14. Арифметические действия над десятичными дробями.
    15. Проценты.
    16. Обращение обыкновенной дроби в бесконечную десятичную периодическую дробь.
    17. Обращение бесконечной десятичной периодической дроби в обыкновенную дробь.
    18. Координатная прямая.
    19. Множество рациональных чисел.
    § 3. Действительные числа
    21. Действительные числа. Числовая прямая.
    22 Обозначения некоторых числовых множеств.
    23. Сравнение действительных чисел.
    25. Числовые промежутки.
    26. Модуль действительного числа.
    27. Формула расстояния между двумя точками координатной прямой.
    28. Правила действий над действительными числами.
    29. Свойства арифметических действий над действительными числами.
    30. Пропорции.
    31. Целая часть числа. Дробная часть числа.
    32. Степень с натуральным показателем.
    33. Степень с нулевым показателем. Степень с отрицательным целым показателем.
    34. Стандартный вид положительного действительного числа.
    35. Определение арифметического корня.
    36. Корень нечетной степени из отрицательного числа.
    37. Степень с дробным показателем.
    38. Свойства степеней с рациональными показателями.
    39. Приближенные значения чисел. Абсолютная и относительная погрешности.
    40. Десятичные приближения действительного числа по недостатку и по избытку.
    41. Правило извлечения квадратного корня из натурального числа.
    42. Понятие о степени с иррациональным показателем.
    43. Свойства степеней с действительными показателями.
    § 4. Комплексные числа
    45. Арифметические операции над комплексными числами.
    46. Алгебраическая форма комплексного числа.
    47. Отыскание комплексных корней уравнений.
    ГЛАВА II. АЛГЕБРАИЧЕСКИЕ ВЫРАЖЕНИЯ
    49. 3.
    112. Построение графика функции y = f(x-m)+n
    113. График квадратичной функции.
    114. Способы построения графика квадратичной функции
    115. Построение графика функции y = f(kx).
    116. Сжатие и растяжение графиков тригонометрических функций.
    117. График гармонического колебания
    ГЛАВА IV. ТРАНСЦЕНДЕНТНЫЕ ВЫРАЖЕНИЯ
    § 12. Преобразование выражений, содержащих переменную под знаком логарифма
    119. Определение логарифма положительного числа по данному основанию.
    120. Свойства логарифмов.
    121. Переход к новому основанию логарифма.
    122. Логарифмирование и потенцирование.
    123. Десятичный логарифм. Характеристика и мантисса десятичного логарифма.
    § 13. Формулы тригонометрии и их использование для преобразования тригонометрических выражений
    125. Формулы сложения и вычитания аргументов.
    126. Формулы приведения.
    127. Соотношения между тригонометрическими функциями одного и того же аргумента.
    128. Формулы двойного угла.
    129. Формулы понижения степени.
    130. Преобразование суммы тригонометрических функций в произведение.
    131. Преобразование произведения тригонометрических функций в сумму.
    132. Преобразование выражения a cos t + b sin t к виду A sin (t + a).
    133. Примеры преобразований выражений, содержащих обратные тригонометрические функции.
    ГЛАВА V. УРАВНЕНИЯ И СИСТЕМЫ УРАВНЕНИЙ
    § 14. Уравнения с одной переменной
    135. Равносильность уравнений.
    136. Линейные уравнения.
    137. Квадратные уравнения.
    138. Неполные квадратные уравнения.
    139. Теорема Виета.
    140. Системы и совокупности уравнений.
    141. Уравнения, содержащие переменную под знаком модуля.
    142. Понятие следствия уравнения. Посторонние корни.
    143. Уравнения с переменной в знаменателе.
    144. Область определения уравнения.
    145. Рациональные уравнения.
    146. Решение уравнения p(x) = 0 методом разложения его левой части на множители.
    147. Решение уравнений методом введения новой переменной.
    148. Биквадратные уравнения.
    149. Решение задач с помощью составления уравнений.
    150. Иррациональные уравнения.
    151. Показательные уравнения.
    152. Логарифмические уравнения.
    153. Примеры решения показательно-логарифмических уравнений.
    154. Простейшие тригонометрические уравнения.
    155. Методы решения тригонометрических уравнений.
    156. Универсальная подстановка (для тригонометрических уравнений).
    157. Метод введения вспомогательного аргумента (для тригонометрических уравнений).
    158. Графическое решение уравнений.
    159. Уравнения с параметром.
    § 15. Уравнения с двумя переменными
    161. График уравнения с двумя переменными.
    162. Линейное уравнение с двумя переменными и его график.
    § 16. Системы уравнений
    164. Решение систем двух уравнений с двумя переменными методом подстановки.
    165. Решение систем двух уравнений с двумя переменными методом сложения.
    167. Графическое решение систем двух уравнений с двумя переменными.
    168. Исследование системы двух линейных уравнений с двумя переменными.
    169. Решение систем двух уравнений с двумя переменными методами умножения и деления.
    170. Системы показательных и логарифмических уравнений.
    171. Системы тригонометрических уравнений с двумя переменными.
    172. Системы трех уравнений с тремя переменными.
    173. Решение задач с помощью составления систем уравнений.
    Глава VI. НЕРАВЕНСТВА
    § 17. Решение неравенств с переменной
    175. Графическое решение неравенств с одной переменной.
    176. Линейные неравенства с одной переменной.
    177. Системы неравенств с одной переменной.
    178. Совокупность неравенств с одной переменной.
    179. Дробно-линейные неравенства.
    180. Неравенства второй степени.
    181. Графическое решение неравенств второй степени.
    182. Неравенства с модулями.
    183. Решение рациональных неравенств методом промежутков.
    184. Показательные неравенства.
    185. Логарифмические неравенства.
    186. Иррациональные неравенства.
    187. Решение тригонометрических неравенств.
    188. Неравенства и системы неравенств с двумя переменными.
    § 18. Доказательство неравенств
    190. Синтетический метод доказательства неравенств.
    191. Доказательство неравенств методом от противного.
    192. Использование неравенств при решении уравнений.
    ГЛАВА VII. ЭЛЕМЕНТЫ МАТЕМАТИЧЕСКОГО АНАЛИЗА
    § 19. Числовые последовательности
    194. Способы задания последовательности.
    195. Возрастание и убывание последовательности.
    196. Определение арифметической прогрессии.
    197. Свойства арифметической прогрессии
    198. Определение геометрической прогрессии.
    199. Свойства геометрической прогрессии.
    200. Понятие о пределе последовательности.
    201. Вычисление пределов последовательностей.
    202. Сумма бесконечной геометрической прогрессии при |q| § 20. Предел функции
    204. Вычисление пределов функции при х->оо.
    205. Предел функции в точке. Непрерывные функции.
    206. Вертикальная асимптота.
    207. Вычисление пределов функций в точке.
    § 21. Производная и ее применения
    209. Определение производной.
    210. Формулы дифференцирования. Таблица производных.
    211. Дифференцирование суммы, произведения, частного.
    212. Сложная функция и ее дифференцирование.
    213. Физический смысл производной.
    214. Вторая производная и ее физический смысл.
    215. Касательная к графику функции.
    216. Применение производной к исследованию функций на монотонность.
    217. Применение производной к исследованию функций на экстремум.
    218. Отыскание наибольшего и наименьшего значений непрерывной функции на отрезке.
    219. Отыскание наибольшего или наименьшего значения непрерывной функции на незамкнутом промежутке.
    220. Задачи на отыскание наибольших или наименьших значений величин.
    221. Применение производной для доказательства тождеств.
    222. Применение производной для доказательства неравенств.
    223. Общая схема построения графика функции.
    § 22. Первообразная и интеграл
    225. Таблица первообразных.
    226. Правила вычисления первообразных.
    227. Интеграл.
    228. Связь между интегралов и первообразной (формула Ньютона—Лейбница).
    229. Правила вычисления интегралов.
    230. Использование интеграла для вычисления площадей плоских фигур.
    ГЕОМЕТРИЯ. ГЛАВА I. ГЕОМЕТРИЧЕСКИЕ ФИГУРЫ НА ПЛОСКОСТИ
    2. Точка. Прямая.
    3. Определения. Аксиомы. Теоремы.
    § 2. Основные свойства простейших геометрических фигур
    5. Луч.
    6. Окружность. Круг.
    7. Полуплоскость.
    8. Угол. Градусная мера угла.
    9. Смежные и вертикальные углы.
    10. Центральные и вписанные углы.
    11. Параллельные прямые.
    12. Признаки параллельности прямых.
    13. Перпендикулярные прямые.
    14. Касательная к окружности.
    15. Треугольники.
    16. Равенство треугольников.
    17. Равнобедренный треугольник.
    18. Сумма углов треугольника.
    19. Прямоугольный треугольник. Теорема Пифагора.
    20. Окружности, вписанные в треугольник и описанные около треугольника.
    § 3. Геометрические построения на плоскости
    22. Простейшие задачи на построение.
    23. Геометрическое место точек на плоскости.
    § 4. Четырехугольники
    25. Параллелограмм.
    26. Прямоугольник. Ромб. Квадрат.
    27. Трапеция.
    § 5. Многоугольники
    29. Выпуклые многоугольники.
    30. Правильные многоугольники.
    31. Длина окружности.
    § 6. Решение треугольников
    33. Соотношения между сторонами и углами в прямоугольном треугольнике.
    34. Теорема косинусов. Теорема синусов.
    35. Решение треугольников.
    § 7. Площади плоских фигур
    37. Площади многоугольников.
    38. Площади подобных фигур.
    39. Площадь круга.
    ГЛАВА II. Прямые и плоскости в пространстве
    § 9. Параллельность прямых и плоскостей
    42. Параллельность прямой и плоскости.
    43. Параллельные плоскости.
    § 10. Перпендикулярность прямых и плоскостей
    45. Перпендикуляр и наклонная к плоскости.
    46. Перпендикулярность плоскостей.
    ГЛАВА III. ТЕЛА В ПРОСТРАНСТВЕ
    § 11. Многогранники
    48. Многогранные углы. Многогранники.
    49. Призма. Параллелепипед. Куб.
    50. Пираприда.
    51. Правильные многогранники.
    § 12. Тела вращения
    53. Конус.
    54. Шар.
    § 13. Изображение пространственных фигур на плоскости
    56. Ортогональное проектирование.
    57. Геометрическое место точек в пространстве.
    § 14. Объемы тел
    59. Объем параллелепипеда, призмы и пирамиды.
    60. Объем цилиндра и конуса.
    61. Общая формула объемов тел вращения.
    § 15. Площади поверхностей тел
    63. Понятие площади поверхности.
    64. Площади поверхностей тел вращения.
    ГЛАВА IV. ДЕКАРТОВЫ КООРДИНАТЫ
    § 16. Координаты на плоскости и в пространстве
    66. Координаты середины отрезка.
    § 17. Уравнения фигур на плоскости
    68. Пересечение двух окружностей.
    69. Уравнение прямой.
    70. Пересечение прямой и окружности.
    § 18. Уравнения фигур в пространстве
    72. Уравнение сферы.
    73. Взаимное расположение сферы и плоскости.
    74. Пересечение двух сфер.
    ГЛАВА V. РЕОБРАЗОВАНИЯ ФИГУР
    76. Понятие движения.
    § 20. Подобие фигур
    78. Подобные фигуры.
    ГЛАВА VI. ВЕКТОРЫ
    80. Понятие вектора.
    81. Координаты вектора.
    § 22. Операции над векторами
    83. Умножение вектора на число. Коллинеарные векторы.
    84. Скалярное произведение векторов.
    ПРИЛОЖЕНИЯ
    ГЕОМЕТРИЯ

    Расстояние между двумя точками. Середина отрезка. Координаты середины отрезка. Тема 4

    1. Тема 1-11. Расстояние между двумя точками. Середина отрезка. Координаты середины отрезка. Уравнение прямой на плоскости.

    Раздел III. Аналитическая геометрия
    Тема 1-11.
    Расстояние между двумя точками. Середина
    отрезка. Координаты середины отрезка.
    Уравнение прямой на плоскости. Уравнение
    прямой в пространстве. Уравнение
    плоскости. Расстояние от точки до плоскости.
    Расстояние между плоскостями. Расстояние
    от точки до прямой на плоскости. Расстояние
    от точки до прямой в пространстве. Угол
    между плоскостями. Угол между прямой и
    плоскостью

    2. Расстояние между двумя точками — это длина отрезка, что соединяет эти точки.

    Расстояние между двумя точками — это
    длина отрезка, что соединяет эти точки.
    • Формула вычисления расстояния между двумя
    точками A(xa, ya) и B(xb, yb) на плоскости:
    AB = √(xb — xa)2 + (yb — ya)2
    • Формула вычисления расстояния между двумя
    точками A(xa, ya, za) и B(xb, yb, zb) в пространстве:
    AB = √(xb — xa)2 + (yb — ya)2 + (zb — za)2

    3. Середина отрезка — это точка, которая лежит на отрезке и находится на равном расстоянии от конечных точек.

    Середина отрезка — это точка, которая
    лежит на отрезке и находится на равном
    расстоянии от конечных точек.
    • Формула вычисления координат середины
    отрезка с концами A(xa, ya) и B(xb, yb) на
    плоскости:
    x a + xb
    ya + yb
    xc =
    yc =
    2
    2
    • Формула вычисления координат середины
    отрезка с концами A(xa, ya, za) и B(xb, yb, zb) в
    пространстве:
    xa + xb
    ya + yb
    za + zb
    xc =
    yc =
    zc =
    2
    2
    2

    4.

    Прямая (прямая линия) — это бесконечная линия, по которой проходит кратчайший путь между любыми двумя её точками.Прямая (прямая линия) — это
    бесконечная линия, по которой
    проходит кратчайший путь между
    любыми двумя её точками.
    • Любую прямую на плоскости можно
    задать уравнением прямой первой степени
    вида
    A x + B y + C = 0,
    где A и B не могут быть одновременно равны
    нулю.

    5. Уравнение прямой с угловым коэффициентом

    • Общее уравнение прямой при B≠0 можно
    привести к виду
    y = k x + b,
    где k — угловой коэффициент равный тангенсу
    угла, образованного данной прямой и
    положительным направлением оси ОХ.

    6. Уравнение прямой в отрезках на осях

    • Если прямая пересекает оси OX и OY в
    точках с координатами (a, 0) и (0, b), то она
    может быть найдена используя
    формулу уравнения прямой в отрезках
    x
    a
    +
    y
    b
    =1

    7. Уравнение прямой, проходящей через две различные точки на плоскости

    • Если прямая проходит через две точки
    A(x1, y1) и B(x2, y2), такие
    что x1 ≠ x2 и y1 ≠ y2 то уравнение
    прямой можно найти, используя
    следующую формулу
    x -x1
    x2 -x1
    =
    y -y1
    y2 — y1

    8.

    Параметрическое уравнение прямой на плоскости• Параметрические уравнения прямой могут
    быть записаны следующим образом
    x = l t +x0
    y = m t + y0
    где (x0, y0) — координаты точки лежащей на
    прямой,
    {l,m} — координаты направляющего вектора
    прямой.

    9. Каноническое уравнение прямой на плоскости

    • Если известны координаты точки A(x0, y0)
    лежащей на прямой и направляющего
    вектора n ={l;m}, то уравнение прямой
    можно записать в каноническом виде,
    используя следующую формулу
    x -x0
    l
    =
    y — y0
    m

    10. Уравнение прямой, проходящей через две различные точки в пространстве

    • Если прямая проходит через две точки
    A(x1,y1,z1) и B(x2,y2,z2), такие что
    x1 ≠ x2, y1 ≠ y2 и z1 ≠ z2 то уравнение
    прямой можно найти используя
    следующую формулу
    x -x1
    x2 -x1
    =
    y -y1
    y2 -y1
    =
    z -z1
    z2 — z1

    11. Параметрическое уравнение прямой в пространстве

    • Параметрические уравнения прямой могут
    быть записаны следующим образом
    x = l t +x0
    y = m t + y0
    z = n t + z0
    где (x0, y0, z0) — координаты точки лежащей на
    прямой,
    {l; m; n} — координаты направляющего вектора
    прямой.

    12. Каноническое уравнение прямой в пространстве

    • Если известны координаты точки A(x0, y0, z0)
    лежащей на прямой и направляющего
    вектора n={l;m;n}, то уравнение прямой
    можно записать в каноническом виде,
    используя следующую формулу
    x -x0
    l
    =
    y -y0
    m
    =
    z -z0
    n

    13. Прямая как линия пересечения двух плоскостей

    • Если прямая является пересечением двух
    плоскостей, то ее уравнение можно задать
    следующей системой уравнений
    A1x + B1y + C1z + D1 = 0
    A2x + B2y + C2z + D2 = 0
    при условии, что не имеет место равенство
    A1
    A2
    =
    B1
    B2
    =
    C1
    C2
    .

    14. Плоскость — есть поверхность, полностью содержащая, каждую прямую, соединяющую любые её точки.

    Плоскость — есть поверхность, полностью
    содержащая, каждую прямую,
    соединяющую любые её точки.
    • Любую плоскость можно
    задать уравнением плоскости первой
    степени вида
    Ax+By+Cz+D=0
    где A, B и C не могут быть одновременно
    равны нулю.

    15. Уравнение плоскости в отрезках

    • Если плоскость пересекает оси OX, OY и OZ в
    точках с координатами (a, 0, 0), (0, b, 0) и (0,
    0, с), то она может быть найдена, используя
    формулу уравнения плоскости в отрезках
    x
    a
    +
    y
    b
    +
    z
    c
    =1

    16. Уравнение плоскости, проходящей через точку, перпендикулярно вектору нормали

    • Чтобы составить уравнение плоскости, зная
    координаты точки плоскости M(x0, y0, z0) и
    вектора нормали плоскости n = {A; B; C} можно
    использовать следующую формулу.
    A(x — x0) + B(y — y0) + C(z — z0) = 0

    17. Уравнение плоскости, проходящей через три заданные точки, не лежащие на одной прямой

    • Если заданы координаты трех точек
    A(x1, y1, z1), B(x2, y2, z2) и C(x3, y3, z3),
    лежащих на плоскости, то уравнение
    плоскости можно найти по следующей
    формуле
    x — x1 y — y1 z — z1
    x2 — x1 y2 — y1 z2 — z1
    x3 — x1 y3 — y1 z3 — z1
    =0

    18. Расстояние от точки до плоскости — равно длине перпендикуляра, опущенного из точки на плоскость.

    Расстояние от точки до плоскости —
    равно длине перпендикуляра,
    опущенного из точки на плоскость.
    • Если задано уравнение плоскости Ax + By +
    Cz + D = 0, то расстояние от точки M(Mx, My,
    Mz) до плоскости можно найти, используя
    следующую формулу:
    |A·Mx + B·My + C·Mz + D|
    d=
    √A2 + B2 + C2

    19. Расстояние между плоскостями — равно длине перпендикуляра, опущенного с одной плоскости на другую.

    Расстояние между плоскостями — равно
    длине перпендикуляра, опущенного с
    одной плоскости на другую.
    • Если заданы уравнения параллельных
    плоскостей Ax + By + Cz + D1 = 0 и
    Ax + By + Cz + D2 = 0, то расстояние между
    плоскостями можно найти, используя
    следующую формулу
    |D2 — D1|
    d=
    √A2 + B2 + C2

    20. Расстояние от точки до прямой — равно длине перпендикуляра, опущенного из точки на прямую.

    Расстояние от точки до прямой — равно
    длине перпендикуляра, опущенного из
    точки на прямую.
    • Если задано уравнение прямой Ax + By + C = 0,
    то расстояние от точки M(Mx, My) до прямой
    можно найти, используя следующую формулу
    |A·Mx + B·My + C|
    d=
    √A2 + B2

    21.

    Расстояние от точки до прямой — равно длине перпендикуляра, опущенного из точки на прямую.Расстояние от точки до прямой —
    равно длине перпендикуляра,
    опущенного из точки на прямую.
    • Если s = {m; n; p} — направляющий вектор
    прямой l, M1(x1, y1, z1) — точка лежащей на
    прямой, тогда расстояние от точки
    M0(x0, y0, z0) до прямой l можно найти,
    используя формулу
    d=
    |M0M1×s|
    |s|
    • Двугранный угол между плоскостями равен углу
    образованному нормальными векторами этих
    плоскостей.
    • Двугранный угол между плоскостями равен углу
    образованному прямыми l1 и l2, лежащими в
    соответствующих плоскостях и перпендикулярными
    линии пересечения плоскостей.
    • Если заданы уравнения плоскостей A1x + B1y + C1z +
    D1 = 0 и A2x + B2y + C2z + D2 = 0, то угол между
    плоскостями можно найти, используя следующую
    формулу
    |A1·A2 + B1·B2 + C1·C2|
    cos α =
    √A12 + B12 + C12√A22 + B22 + C22

    23. Угол между прямой и плоскостью — это угол между прямой и ее проекцией на эту плоскость.

    Угол между прямой и плоскостью — это угол
    между прямой и ее проекцией на эту
    плоскость.
    • Если в пространстве заданы направляющий
    вектор прямой L s = {l; m; n} и уравнение
    плоскости Ax + By + Cz + D = 0, то угол
    между этой прямой и плоскостью можно
    найти используя формулу
    |A·l+B· m+C· n|
    sin φ =
    √A2 + B2 + C2 · √l2 + m2 + n2

    Пояснение к уроку: Точки, средние точки и расстояния в пространстве

    В этом объяснении мы научимся найдите координаты точки в 3D, расстояние между двумя точками в 3D, а также координаты средней и конечной точек в 3D, используя формулу.

    Мы уже должны знать, как найти все это в двух измерениях. Любая точка в двух измерениях будет иметь 𝑥- и 𝑦-координату и может быть записана в виде (𝑥,𝑦). Каждое из действительных чисел в упорядоченной паре представляет собой смещение этой точки от начала координат, другими словами, положительное или отрицательное расстояние от точки (0,0).

    Если две точки 𝐴 и 𝐵 имеют координаты (𝑥,𝑦) и (𝑥,𝑦) соответственно, то мы можем вычислить их середину по формуле 𝑥+𝑥2,𝑦+𝑦2.

    Если две точки 𝐴 и 𝐵 имеют координаты (𝑥,𝑦) и (𝑥,𝑦) соответственно, то мы можем вычислить расстояние между ними, используя формулу расстояния, полученную из теоремы Пифагора, (𝑥−𝑥)+(𝑦−𝑦).

    В этом объяснении мы исследуем, как мы можем расширить эти формулы, чтобы включить третью координату при работе с точками в трех измерениях.

    Определение: Координаты точки в трехмерном пространстве

    Любая точка в трех измерениях будет иметь 𝑥-, 𝑦-, и 𝑧-координат и может быть записан в виде (𝑥,𝑦,𝑧). Каждое из действительных чисел в упорядоченной тройке дает расстояние от начала координат, измеренное вдоль соответствующей оси.

    В нашем первом примере мы рассмотрим, в какой плоскости лежит точка, одна из координат которой равна нулю.

    Пример 1. Определение плоскости, в которой находится заданная координата

    В какой из следующих координатных плоскостей находится точка (−7,−8,0) ложь?

    1. 𝑥𝑦
    2. 𝑥𝑧
    3. 𝑦𝑧

    Ответ

    Мы знаем, что точка в 3D будет иметь 𝑥-, 𝑦- и 𝑧-координаты. В этом вопросе 𝑥=−7, 𝑦=−8 и 𝑧=0.

    Поскольку 𝑧-координата равна нулю, точка находится на нулевом расстоянии от начала координат в 𝑧-направлении. Это означает, что он будет лежать на 𝑥𝑦-плоскости. В самом деле, любая точка с координаты (𝑥,𝑦,0) будут лежать на этой плоскости.

    Таким образом, мы можем заключить, что точка (−7,−8,0) лежит на 𝑥𝑦-плоскости.

    Определение: Три координатные плоскости

    Любая точка с координатами (𝑥,𝑦,0) будет лежать на 𝑥𝑦-плоскости.

    Аналогично, любая точка с координатами (𝑥,0,𝑧) будет лежать на 𝑥𝑧-плоскость, и любая точка с координатами (0,𝑦,𝑧) будет лежать на 𝑦𝑧-плоскости.

    В нашем следующем вопросе мы рассмотрим, как мы можем определить координаты точки в трех измерениях.

    Пример 2. Нахождение координат заданной точки в 3D

    Определить координаты точки 𝐴.

    Ответ

    Любая точка в трех измерениях будет иметь 𝑥-, 𝑦- и 𝑧-координаты и может быть записана в виде (𝑥,𝑦,𝑧).

    Двигаясь от начала координат, мы проходим 3 единицы в положительном 𝑥-направлении, −3 единицы в 𝑦-направлении и, наконец, 3 единицы в 𝑧-направлении.

    Это означает, что 𝑥=3, 𝑦=−3 и 𝑧=3.

    Координаты точки 𝐴 равны (3,−3,3).

    Напомним, что формула средней точки в двух измерениях просто говорит нам найти среднее значение двух точек. Находим среднее значение 𝑥-координат и среднее значение 𝑦-координат. Теперь мы расширим эту идею на три измерения, найдя также среднее значение 𝑧-координат.

    Чтобы найти среднее любых двух чисел, мы складываем их, а затем делим их сумму на два.

    Определение: середина двух точек в трехмерном пространстве

    Если две точки 𝐴 и 𝐵 имеют координаты (𝑥,𝑦,𝑧) и (𝑥,𝑦,𝑧) соответственно, то можно вычислить их среднюю точку, используя следующую формулу: 𝑥+𝑥2,𝑦+𝑦2,𝑧+𝑧2.

    В нашем следующем примере мы будем использовать эту формулу для определения середины двух точек в пространстве.

    Пример 3: Нахождение координат средней точки в 3D

    Баллы 𝐴 и 𝐵 имеют координаты (8,−8,−12) и (−8,5,−8) соответственно. Определить координаты середины 𝐴𝐵.

    Ответ

    Чтобы найти середину двух точек в трех измерениях, мы будем использовать формулу для вычисления середины координат (𝑥,𝑦,𝑧) и (𝑥,𝑦,𝑧): 𝑥+𝑥2,𝑦+𝑦2,𝑧+𝑧2. 

    Пусть точка 𝐴 имеет координаты (𝑥,𝑦,𝑧), а точка 𝐵 имеет координаты (𝑥 ,𝑦,𝑧 ).

    Середина между точками 𝐴 и 𝐵 равна =8+(−8)2,−8+52,−12+(−8)2=02,−32,−202=0,−32 ,−10.

    Координаты середины 𝐴𝐵 составляют 0,−32,−10.

    В нашем следующем примере мы будем использовать формулу средней точки для определения конечной точки по средней точке двух точек в пространстве и другой конечной точке.

    Пример 4: Нахождение координат конечной точки отрезка линии по координатам середины и координатам начальной точки

    Учитывая, что точка (0,17,−10) является средней точкой 𝐴𝐵 и это 𝐴(−19,7,14), каковы координаты 𝐵?

    Ответ

    Чтобы найти середину двух точек в трех измерениях, мы будем использовать формулу для вычисления середины координат (𝑥,𝑦,𝑧) и (𝑥,𝑦,𝑧) : 𝑥+𝑥2,𝑦+𝑦2,𝑧+𝑧2.

    Мы знаем, что точка 𝐴 имеет координаты (−19,7,14) и пусть точка 𝐵 имеет координаты (𝑥,𝑦,𝑧). Середина между этими двумя точками имеет координаты (0,17,−10).

    Подставив эти значения в формулу, получим (0,17,−10)=−19+𝑥2,7+𝑦2,14+𝑧2.

    Затем мы можем приравнять отдельные компоненты, что даст нам решение трех уравнений.

    Во-первых, 𝑥-координата дает нам 0=−19+𝑥2.

    Умножая обе части уравнения на 2, получаем 0=−19+𝑥.

    Итак, 19=𝑥.

    Во-вторых, 𝑦-координата дает нам 17=7+𝑦2.

    Умножив обе части уравнения на 2, мы получим 34=7+𝑦.

    Итак, 27=𝑦.

    Наконец, 𝑧-координата дает нам −10=14+𝑧2.

    Умножая обе части уравнения на 2, мы получаем −20=14+𝑧.

    Итак, −34=𝑧.

    Координаты точки 𝐵 равны (19,27,−34).

    В двух измерениях мы можем вычислить расстояние между двумя точками, используя адаптацию теоремы Пифагора. Это утверждает, что 𝑎+𝑏=𝑐, где 𝑐 — длина самой длинной стороны, известной как гипотенуза, прямоугольного треугольника.

    Если две точки 𝐴 и 𝐵 имеют координаты (𝑥,𝑦) и (𝑥,𝑦) соответственно, то можно вычислить расстояние между ними по следующей формуле: (𝑥−𝑥)+(𝑦−𝑦). 

    Теперь мы рассмотрим, как мы можем вычислить расстояние между двумя точками в три измерения.

    Рассмотрим трехмерную прямоугольную призму 𝐴𝐵𝐶𝐷𝐸𝐹𝐺𝐻, нарисованную ниже, и предположим, что мы хотим пройти от самого нижнего левого переднего угла, 𝐴, до самого верхнего правого заднего угла, 𝐺.

    Сначала рассмотрим треугольник 𝐴𝐵𝐹 в основании призмы. Теорема Пифагора говорит нам, что 𝐴𝐹=𝐴𝐵+𝐵𝐹.

    Итак, 𝐴𝐹=√𝑥+𝑦.

    Теперь делаем еще один треугольник 𝐴𝐹𝐺, с основанием вдоль 𝐴𝐹 и высотой 𝐹𝐺.

    Мы можем снова использовать теорему Пифагора так, чтобы 𝐴𝐺=𝐴𝐹+𝐹𝐺. Подставляя длины 𝐴𝐹 и 𝐹𝐺, мы видим, что 𝐴𝐺=√𝑥+𝑦+𝑧.

    Следовательно, 𝐴𝐺=√𝑥+𝑦+𝑧.

    Определение: расстояние между двумя точками в трехмерном пространстве

    Если две точки 𝐴 и 𝐵 имеют координаты (𝑥,𝑦,𝑧) и (𝑥,𝑦,𝑧), соответственно, то мы можем рассчитать расстояние между ними по следующей формуле: (𝑥−𝑥)+(𝑦−𝑦)+(𝑧−𝑧). теоремы Пифагора в трех измерениях; мы находим сумму квадратов разницы между каждой координатой и затем квадратный корень этого ответа.

    В наших последних двух вопросах мы вычислим кратчайшее расстояние между точкой и одной из осей, а также расстояние между двумя точками в пространстве.

    Пример 5. Нахождение расстояния между двумя точками по их координатам в трех измерениях

    Нахождение расстояния между двумя точками 𝐴(−7,12,3) и 𝐵(−4,−1,−8).

    Ответ

    Чтобы вычислить расстояние между двумя точками в трех измерениях, мы будем использовать следующую формулу, где две точки 𝐴 и 𝐵 имеют координаты (𝑥,𝑦,𝑧) и (𝑥,𝑦,𝑧) соответственно: (𝑥−𝑥)+(𝑦−𝑦)+(𝑧−𝑧).

    Пусть точка 𝐴 имеет координаты (𝑥,𝑦,𝑧), а точка 𝐵 имеет координаты (𝑥,𝑦,𝑧).

    Расстояние между ними равно =√(−4−(−7))+(−1−12)+(−8−3)=√(3)+(−13)+(−11)=√9+169+121=√299 .

    Расстояние между двумя точками 𝐴(−7,12,3) и 𝐵(−4,−1,−8) составляет √299 единиц длины.

    Пример 6: Нахождение расстояния между точкой и осью в 3D

    Каково расстояние между точкой (19,5,5) и осью 𝑥?

    Ответ

    Мы знаем, что любая точка будет лежать на оси 𝑥, если и ее 𝑦-координата, и ее 𝑧-координата равны нулю. Это означает, что мы можем определить точку на оси 𝑥 как (𝑥,0,0).

    Признаем, что искомое расстояние — это перпендикулярное расстояние от точки до оси 𝑥, что означает проекцию точки на 𝑥-ось будет в точке (19,0,0).

    Расстояние между двумя точками можно рассчитать по формуле следующим образом √(19−19)+(5−0)+(5−0)=√0+(5)+(5)=√50=5√2.

    Расстояние между точка (19,5,5) и ось 𝑥 составляют 5√2 единиц длины.

    Мы закончим это объяснение повторением некоторых ключевых моментов.

    Ключевые точки

    • Любая точка в трех измерениях имеет координаты, записанные в виде (𝑥,𝑦,𝑧).
    • Если 𝑧-координата равна нулю, то мы знаем, что точка лежит в 𝑥𝑦-плоскости; если 𝑦-координата равна нулю, то мы знаем, что точка лежит в 𝑥𝑧-плоскость; а если 𝑥-координата равна нулю, то мы знаем, что точка лежит в 𝑦𝑧-плоскости.
    • Если и 𝑦-координата, и 𝑧-координата равны нулю, то точка лежит на 𝑥-оси; если и 𝑥-координата, и 𝑧-координата равна нулю, то точка лежит на 𝑦-оси; и если и 𝑥-координата, и 𝑦-координата равны равна нулю, то точка лежит на оси 𝑧.
    • Середина двух точек с координатами (𝑥,𝑦,𝑧) и (𝑥,𝑦,𝑧) лежит в точке 𝑥+𝑥2,𝑦+𝑦2,𝑧+𝑧2.
    • Мы также можем использовать формулу средней точки для вычисления конечной точки отрезка, учитывая среднюю точку и другую конечную точку.
    • Расстояние между двумя точками с координатами (𝑥,𝑦,𝑧) и (𝑥,𝑦,𝑧) это равно (𝑥−𝑥)+(𝑦−𝑦)+(𝑧−𝑧).

    Центр отрезка.

    Произношение: /ˈmɪdˌpɔɪnt/ Объяснение

    Середина — это точка равноудаленный между двумя точками. Середина находится на отрезке, соединяющем две точки и делит отрезок ровно пополам. Точное математическое определение средней точки:

    Средняя точка A M между точками А и В точка на прямой AB такая, что АМ = МБ .

    Как построить среднюю точку

    Шаг Иллюстрация Описание
    1 Начните с точек A и Б .
    2 Нарисуйте отрезок АВ.
    3 Нарисуйте круг с центром в точке А и радиус AB .
    4 Нарисуйте круг с центром в точке B и радиус AB
    5 Отметить одно пересечение двух кругов как точку C и другой перекресток двух круги как точка D .
    6 Нарисуйте отрезок CD.
    7 Отметить точку пересечения сегмента линии АВ и отрезок CD как М . Точка M — средняя точка.
    Таблица 1: Построение средней точки.

    Как вычислить среднюю точку в одномерном метрическом пространстве

    Нажмите на синие точки и перетащите их, чтобы изменить фигуру.

    Что произойдет, если B окажется слева от A?
    Манипулятивное 8 — Расчет средней точки в одном измерении Создано с помощью GeoGebra.

    Формула средней точки в одномерном пространстве между А и В есть . Нажать на синие точки в манипуляции 1 и перетащите их, чтобы изменить фигуру.

    Как вычислить среднюю точку в метрическом двумерном пространстве

    Нажмите на синие точки и перетащите их, чтобы изменить фигуру.

    Манипулятивное 9 — Расчет средней точки в двух измерениях Создано с помощью GeoGebra.

    Середина делит отрезок ровно пополам. Этот факт можно использовать найти формулу для середины отрезка в двумерном пространстве. Евклидово пространство, такое как декартова система координат. X-координата средняя точка будет на полпути между x-координатами двух точек, и y-координата средней точки будет на полпути между y-координатами две точки. Формула середины отрезка с концами и является .

    Как вычислить среднюю точку в метрическом n-мерном пространстве

    Алгоритм вычисления конечной точки в двумерном пространство может быть обобщено для n-мерного пространства. Учитывая две точки и середина .

    Доказательство: Если

    M является средней точкой АБ, затем утра = MB

    Это доказательство является доказательством абзаца или неофициальным доказательством.

    Определение середины отрезка таково, что 902:01 AM = МБ . Другими словами, длины двух отрезков равны. По определению конгруэнтности АМ конгруэнтно МБ тогда и только тогда, когда AM и MB имеют одинаковую меру. Так как по определению середины АМ и МБ имеют одинаковую меру, утра = МБ.

    Ссылки

    1. МакАдамс, Дэвид Э.. Словарь всех математических слов, середина . 2-й классный выпуск 20150108-4799968. стр. 117. Life is a Story Problem LLC. 8 января 2015. Купить книгу

    Дополнительная информация

    • Евклид Александрийский. Элементы . Университет Кларка. 06.09.2018. https://mathcs.clarku.edu/~djoyce/elements/elements.html.

    Цитируйте эту статью как:

    МакАдамс, Дэвид Э. Середина . 25.04.2019. Вся энциклопедия математических слов. ООО «Жизнь — это проблема истории». https://www.allmathwords.org/en/m/midpoint.html.

    Авторы изображений

    • Все изображения и манипуляции принадлежат Дэвиду МакАдамсу, если не указано иное. Все изображения Дэвида МакАдамса защищены авторским правом © Life is a Story Problem LLC и находятся под лицензией Creative Commons Attribution-ShareAlike 4.0 International License.

    История изменений

    25.

    Квантили нормального распределения таблица: Таблица. Функция распределения вероятностей стандартного нормального закона. Таблица квантилей стандартного нормального закона распределения.

    Таблица. Функция распределения вероятностей стандартного нормального закона. Таблица квантилей стандартного нормального закона распределения.

    Раздел недели: Скоропись физического, математического, химического и, в целом, научного текста, математические обозначения. Математический, Физический алфавит, Научный алфавит.


    Поиск на сайте DPVA

    Поставщики оборудования

    Полезные ссылки

    О проекте

    Обратная связь

    Ответы на вопросы.

    Оглавление

    Таблицы DPVA.ru — Инженерный Справочник



    Адрес этой страницы (вложенность) в справочнике dpva.ru:  главная страница / / Техническая информация/ / Математический справочник / / Теория вероятностей. Математическая статистика. Комбинаторика. / / Таблица. Функция распределения вероятностей стандартного нормального закона. Таблица квантилей стандартного нормального закона распределения.

    Поделиться:   

    Таблица. Функция распределения вероятностей стандартного нормального закона.

    • Пример использования: на пересечении строки 1.3 и столбца 0,02 находим Ф(1,32)= 0,9049
    • Посмотреть: Таблица квантилей стандартного нормального закона распределения.
    Таблица. Функция распределения стандартного нормального закона.

    t

    . 00

    .01

    .02

    .03

    .04

    .05

    .06

    .07

    .08

    .09

    0

    .5000 .5040 .5080 .5120 .5160 .5199 .5239 .5279 .5319 .5359

    .1

    .5398 .5438 .5478 .5517 .5557 . 5596 .5636 .5675 .5714 .5753

    .2

    .5793 .5832 .5871 .5910 .5948 .5987 .6026 .6064 .6103 .6141

    .3

    .6179 .6217 .6255 .6293 .6331 .6368 .6406 .6443 .6480 .6517

    .4

    .6554 .6591 .6628 .6664 .6700 .6736 .6772 .6808 . 6844 .6879

    .5

    .6915 .6950 .6985 .7019 .7054 .7088 .7123 .7157 .7190 .7224

    .6

    .7257 .7291 .7324 .7357 .7389 .7422 .7454 .7486 .7517 .7549

    .7

    .7580 .7611 .7642 .7673 .7704 .7734 .7764 .7794 .7823 .7852

    . 8

    .7881 .7910 .7939 .7967 .7995 .8023 .8051 .8078 .8106 .8133

    .9

    .8159 .8186 .8212 .8238 .8264 .8289 .8315 .8340 .8365 .8389

    1.0

    .8413 .8438 .8461 .8485 .8508 .8531 .8554 .8577 .8599 .8621

    1.1

    .8643 .8665 . 8686 .8708 .8729 .8749 .8770 .8790 .8810 .8830

    1.2

    .8849 .8869 .8888 .8907 .8925 .8944 .8962 .8980 .8997 .9015

    1.3

    .9032 .9049 .9066 .9082 .9099 .9115 .9131 .9147 .9162 .9177

    1.4

    .9192 .9207 .9222 .9236 .9251 . 9265 .9279 .9292 .9306 .9319

    1.5

    .9332 .9345 .9357 .9370 .9382 .9394 .9406 .9418 .9429 .9441

    1.6

    .9452 .9463 .9474 .9484 .9495 .9505 .9515 .9525 .9535 .9545

    1.7

    .9554 .9564 .9573 .9582 .9591 .9599 .9608 .9616 . 9625 .9633

    1.8

    .9641 .9649 .9656 .9664 .9671 .9678 .9686 .9693 .9699 .9706

    1.9

    .9713 .9719 .9726 .9732 .9738 .9744 .9750 .9756 .9761 .9767

    2.0

    .9772 .9778 .9783 .9788 .9793 .9798 .9803 .9808 .9812 .9817

    2. 1

    .9821 .9826 .9830 .9834 .9838 .9842 .9846 .9850 .9854 .9857

    2.2

    .9861 .9864 .9868 .9871 .9875 .9878 .9881 .9884 .9887 .9890

    2.3

    .9893 .9896 .9898 .9901 .9904 .9906 .9909 .9911 .9913 .9916

    2.4

    .9918 .9920 . 9922 .9925 .9927 .9929 .9931 .9932 .9934 .9936

    2.5

    .9938 .9940 .9941 .9943 .9945 .9946 .9948 .9949 .9951 .9952

    2.6

    .9953 .9955 .9956 .9957 .9959 .9960 .9961 .9962 .9963 .9964

    2.7

    .9965 .9966 .9967 .9968 .9969 . 9970 .9971 .9972 .9973 .9974

    2.8

    .9974 .9975 .9976 .9977 .9977 .9978 .9979 .9979 .9980 .9981

    2.9

    .9981 .9982 .9982 .9983 .9984 .9984 .9985 .9985 .9986 .9986

    3.0

    .9987 .9987 .9987 .9988 .9988 .9989 .9989 .9989 . 9990 .9990

    3.1

    .9990 .9991 .9991 .9991 .9992 .9992 .9992 .9992 .9993 .9993

    3.2

    .9993 .9993 .9994 .9994 .9994 .9994 .9994 .9995 .9995 .9995

    3.3

    .9995 .9995 .9995 .9996 .9996 .9996 .9996 .9996 .9996 .9997

    3. 4

    .9997 .9997 .9997 .9997 .9997 .9997 .9997 .9997 .9997 .9998

    Таблица квантилей стандартного нормального закона распределения. Квантили обозначены : Ф(uα)=α

    • Посмотреть:Таблица. Функция распределения стандартного нормального закона.
    Таблица квантилей стандартного нормального закона распределения.

    ?

    uα

    ?

    uα

    ?

    uα

    . 50

    0

    .91

    1.341

    .995

    2.576

    .55

    .126

    .92

    1.405

    .999

    3.090

    .60

    .253

    .93

    1.476

    .9995

    3.291

    .65

    .385

    .94

    1.555

    .9999

    3.719

    . 70

    .524

    .95

    1.645

    .99995

    3.891

    .75

    .674

    .96

    1.751

    .99999

    4.265

    .80

    .842

    .97

    1.881

    .999995

    4.417

    .85

    1.036

    .98

    2.054

    .999999

    4.753

    . 90

    1.282

    .99

    2.326

    .9999999

    5.199

    Поиск в инженерном справочнике DPVA. Введите свой запрос:

    Дополнительная информация от Инженерного cправочника DPVA, а именно — другие подразделы данного раздела:

    Поиск в инженерном справочнике DPVA. Введите свой запрос:

    Если Вы не обнаружили себя в списке поставщиков, заметили ошибку, или у Вас есть дополнительные численные данные для коллег по теме, сообщите , пожалуйста.
    Вложите в письмо ссылку на страницу с ошибкой, пожалуйста.

    Коды баннеров проекта DPVA. ru
    Начинка: KJR Publisiers

    Консультации и техническая
    поддержка сайта: Zavarka Team

    Проект является некоммерческим. Информация, представленная на сайте, не является официальной и предоставлена только в целях ознакомления. Владельцы сайта www.dpva.ru не несут никакой ответственности за риски, связанные с использованием информации, полученной с этого интернет-ресурса. Free xml sitemap generator

    Прикладная статистика: Исследование зависимостей

    Прикладная статистика: Исследование зависимостей
      

    Айвазян С. А. и др. Прикладная статистика: Исследование зависимостей: Справ. изд. / С. А. Айвазян, И. С. Енюков, Л. Д. Мешалкин; Под ред. С. А. Айвазяна. — М.: Финансы и статистика, 1985. — 487 с.

    Данная книга является логическим продолжением справочного издания «Прикладная статистика: Основы моделирования и первичная обработка данных», вышедшего в 1983 г. В ней рассматриваются методы корреляционного, регрессионного и дисперсионного анализа. Приводятся их алгоритмы и обзор программного обеспечения.

    Для статистиков, экономистов, социологов, программистов.



    Оглавление

    ПРЕДИСЛОВИЕ
    Введение. СТАТИСТИЧЕСКОЕ ИССЛЕДОВАНИЕ ЗАВИСИМОСТЕЙ СОДЕРЖАНИЕ, ЗАДАЧИ, ОБЛАСТИ ПРИМЕНЕНИЯ
    В.2. Какова конечная прикладная цель статистического исследования зависимостей?
    В.3. Математический инструментарий
    В.4. Некоторые типовые задачи практики
    В.5. Основные типы зависимостей между количественными переменными
    В.6. Основные этапы статистического исследования зависимостей
    ВЫВОДЫ
    Раздел I. АНАЛИЗ СТРУКТУРЫ И ТЕСНОТЫ СТАТИСТИЧЕСКОЙ СВЯЗИ МЕЖДУ ИССЛЕДУЕМЫМИ ПЕРЕМЕННЫМИ (корреляционный анализ)
    1. 1.1. Понятие индекса корреляции.
    1.1.2. Коэффициент корреляции как измеритель степени тесноты связи в двумерных нормальных схемах.
    1.1.3. Распределение выборочного коэффициента корреляции и проверка гипотезы о статистической значимости линейной связи.
    1.1.4. Влияние ошибок измерения на величину коэффициента корреляции.
    1.1.5. Измерение степени тесноты связи при нелинейной зависимости.
    1.2. Анализ частных («очищенных») связей
    1.2.2. Частные коэффициенты корреляции и их выборочные значения.
    1.2.3. Статистические свойства выборочных частных коэффициентов корреляции (проверка на статистическую значимость их отличия от нуля, доверительные интервалы).
    1.3. Анализ множественных связей
    1.3.2. Множественный коэффициент корреляции и его свойства (общий случай).
    1.3.3. Вычисление и свойства множественного коэффициента корреляции в рамках линейных нормальных моделей.
    1.3.4. Примеры.
    ВЫВОДЫ
    Глава 2. АНАЛИЗ СТАТИСТИЧЕСКОЙ СВЯЗИ МЕЖДУ ПОРЯДКОВЫМИ (ОРДИНАЛЬНЫМИ) ПЕРЕМЕННЫМИ
    2. 1. Ранговая корреляция
    2.1.2. Понятие ранговой корреляции.
    2.1.3. Основные задачи статистического анализа связей между ранжировками.
    2.1.4. Вероятностные пространства ранжировок, генерируемые порядковыми переменными [14, гл. 4, 5].
    2.2. Анализ и измерение парных ранговых статистических связей
    2.2.1. Ранговый коэффициент корреляции Спирмэна.
    2.2.2. Ранговый коэффициент корреляции Кендалла.
    2.2.3. Обобщенная формула для парного коэффициента корреляции и связь между коэффициентами Спирмэна и Кендалла.
    2.2.4. Статистические свойства выборочных характеристик парной ранговой связи.
    2.3. Анализ множественных ранговых связей
    2.3.2. Проверка статистической значимости выборочного значения коэффициента конкордации.
    2.3.3. Использование коэффициента конкордации в решении основных задач статистического анализа ранговых связей.
    2.3.4. Примеры.
    ВЫВОДЫ
    Глава 3. АНАЛИЗ СВЯЗЕЙ МЕЖДУ КЛАССИФИКАЦИОННЫМИ (НОМИНАЛЬНЫМИ) ПЕРЕМЕННЫМИ
    3.1. Таблицы сопряженности
    3. 1.2. Логарифмически-линейная. параметризация таблиц сопряженности.
    3.1.3. Проверка гипотез.
    3.1.4. Меры связи между строками и столбцами таблицы.
    3.2. Приписывание численных значений качественным переменным (дуальное шкалирование)
    3.2.1. Методическое место дуального шкалирования.
    3.2.2. Максимизация F-отношения суммы квадратов отклонений между объектами к полной сумме квадратов отклонений.
    3.2.3. Двойственность в определении V и W.
    3.2.4. Максимизация коэффициента корреляции.
    3.2.5. Изучение оптимального решения.
    3.2.6. Таблицы «объект—многомерный отклик».
    ВЫВОДЫ
    Глава 4. АНАЛИЗ СТРУКТУРЫ СВЯЗЕЙ МЕЖДУ КОМПОНЕНТАМИ МНОГОМЕРНОГО ВЕКТОРА
    4.1.1. Цепи Маркова.
    4.1.3. Математические задачи, связанные с изучением распределений с ДСЗ.
    4.2. Распределение с древообразной структурой зависимостей
    4.2.1. Предварительные сведения из теории графов.
    4.2.2. Распределения с древообразной структурой зависимостей (ДСЗ).
    4.3. Оценка графа структуры зависимостей компонент нормального вектора
    4. 3.2. Построение графа структуры зависимостей по корреляционной матрице.
    4.3.3. Асимптотика Колмогорова — Деева.
    4.4. R(k)-распределения
    4.4.1. Основные определения. Начнем с обобщения понятия распределения с ДСЗ.
    4.4.2. Нормальное R(k)-распределение.
    4.4.3. Восстановление графа структуры зависимостей.
    4.5. Структура связей нормального вектора (общий случай)
    4.5.1. Марковская тройка. Структура многомерного вектора.
    4.5.2. Информационная интерпретация структуры связей.
    4.5.3. Использование структуры для представления распределения в виде композиции более простых распределений.
    ВЫВОДЫ
    Раздел II. ИССЛЕДОВАНИЕ ВИДА ЗАВИСИМОСТИ МЕЖДУ КОЛИЧЕСТВЕННЫМИ ПЕРЕМЕННЫМИ (регрессионный анализ)
    5.1. Функция регрессии как условное среднее и ее интерпретация в рамках многомерной нормальной модели
    5.2. Функция «дельта»-регрессии как решение оптимизационной задачи
    5.3. Взаимоотношения различных регрессий
    ВЫВОДЫ
    Глава 6. ВЫБОР ОБЩЕГО ВИДА ФУНКЦИИ РЕГРЕССИИ
    6. 1. Использование априорной информации о содержательной сущности анализируемой зависимости
    6.2. Предварительный анализ геометрической структуры исходных данных
    6.2.1. Содержание геометрического анализа парных корреляционных полей.
    6.2.2. Учет и формализация «гладких» свойств искомой функции регрессии.
    6.2.3. Некоторые вспомогательные преобразования, линеаризующие исследуемую парную зависимость.
    6.3. Математико-статистические методы в задаче параметризации модели регрессии
    6.3.1. Компромисс между сложностью регрессионной модели и точностью ее оценивания.
    6.3.2. Поиск модели, наиболее устойчивой к варьированию состава выборочных данных, на основании которых она оценивается.
    6.3.3. Статистические критерии проверки гипотез об общем виде функции регрессии.
    ВЫВОДЫ
    Глава 7. ОЦЕНИВАНИЕ НЕИЗВЕСТНЫХ ЗНАЧЕНИЙ ПАРАМЕТРОВ, ЛИНЕЙНО ВХОДЯЩИХ В УРАВНЕНИЕ РЕГРЕССИОННОЙ ЗАВИСИМОСТИ
    7.1. Метод наименьших квадратов
    7.1.2. Свойства мнк-оценок.
    7.1.3. Ортогональная матрица плана.
    7.1.4. Параболическая регрессия и система ортогональных полиномов Чебышева.
    7.1.5. Обобщенный мнк.
    7.2. Функции потерь, отличные от квадратичной
    7.2.1. Функция потерь.
    7.2.3. Функции потерь, имеющие горизонтальную асимптоту.
    7.2.4. Эв-регрессия («лямбда»-регрессия).
    7.2.5. Минимизация систематической ошибки.
    7.3. Байесовское оценивание
    7.3.1. Введение априорной плотности распределения параметров.
    7.3.2. Апостериорное распределение параметров.
    7.3.3. Повторная выборка из той же совокупности.
    7.4. Многомерная регрессия
    7.4.1. Случай известной ковариационной матрицы ошибок.
    7.4.3. Эв-оценки.
    7.4.4. Использование многомерной регрессии для параметризации многомерных распределений.
    7.5. Оценивание параметров при наличии погрешностей в предикторных переменных (конфлюэнтный анализ)
    7.5.1. Основные типы задач конфлюэнтного анализа.
    7.5.2. Модифицированный мнк для схемы активного эксперимента.
    7.5.3. Пассивные наблюдения.
    7.5.4. Некоторые принципиальные отличия регрессионных задач (7.83) и (7.84).
    7.5.5. Неявное задание отклика.
    7.6. Оценивание в регрессионных моделях со случайными параметрами (регрессионные задачи второго рода)
    7.6.2. Случай, когда средние значения и ковариационная матрица оцениваемых параметров известны (требуется оценить параметры).
    7.6.3. Случай, когда известна только ковариационная матрица (требуется оценить параметры).
    7.6.4. Случай неизвестных.
    ВЫВОДЫ
    Глава 8. ОЦЕНИВАНИЕ ПАРАМЕТРОВ РЕГРЕССИИ В УСЛОВИЯХ МУЛЬТИКОЛЛИНЕАРНОСТИ И ОТБОР СУЩЕСТВЕННЫХ ПРЕДИКТОРОВ
    8.1. Явление мультиколлинеарности и его влияние на мнк-оценки
    8.2. Регрессия на главные компоненты
    8.3. Смещенное оценивание коэффициентов регрессии
    8.4. Редуцированные оценки для стандартной модели линейной регрессии
    8.4.2. Редуцированная оценка Мейера — Уилке.
    8.5. Оценки, связанные с ортогональным разложением
    8.5.1. Оптимальное взвешивание вклада главных компонент.
    8. 5.2. Оценка оптимальных вкладов главных компонент.
    8.6. Вопросы точности вычислительной реализации процедур линейного оценивания
    8.6.1. Два метода получения мнк-оценок.
    8.6.2. Оценки величин возмущений для решений центрированной и соответствующей ей нормальной системы уравнений.
    8.6.3. Центрирование и нормирование матрицы данных.
    8.6.4. Вычисление элементов ковариационной матрицы.
    8.7. Отбор существенных переменных в задачах линейной регрессии
    8.7.1. Влияние отбора переменных на оценку уравнения регрессии.
    8.7.2. Критерии качества уравнения регрессии.
    8.7.3. Схемы генерации наборов переменных.
    8.7.4. Пошаговые процедуры генерации наборов.
    8.7.5. Оператор симметричного выметания.
    8.7.6. Методические аспекты использования процедур отбора существенных предикторных переменных.
    ВЫВОДЫ
    Глава 9. ВЫЧИСЛИТЕЛЬНЫЕ АСПЕКТЫ МЕТОДА НАИМЕНЬШИХ КВАДРАТОВ
    9.1. Итерационные методы поиска оценок метода наименьших квадратов (мнк-оценок)
    9.1.2. Алгоритмы квазиградиентного типа.
    9.2. Градиентный спуск
    9.3. Метод Ньютона
    9.4 Метод Ньютона-Гаусса и его модификации
    9.4.2. Обсуждение скорости сходимости процедуры.
    9.4.3. Рекомендации по правилу остановки итерационной процедуры.
    9.5. Методы, не использующие вычисления производных
    9.6. Способы нахождения начального приближения
    9.7. Вопросы существования и единственности мнк-оценки
    ВЫВОДЫ
    Глава 10. НЕПАРАМЕТРИЧЕСКАЯ, ЛОКАЛЬНО-ПАРАМЕТРИЧЕСКАЯ И КУСОЧНАЯ АППРОКСИМАЦИЯ РЕГРЕССИОННЫХ ЗАВИСИМОСТЕЙ
    10.1. Непараметрическое оценивание регрессии
    10.2. Локальная параметрическая аппроксимация регрессии в одномерном случае
    10.3. Кусочно-параметрическая (сплайновая) техника аппроксимации регрессионных зависимостей
    10.3.1. Определение одномерных сплайнов.
    10.3.2. Выбор порядка сплайна, числа и положения узлов.
    10.3.3. Оценка параметров и проверка гипотез.
    10.3.4. Билинейные сплайны.
    ВЫВОДЫ
    Глава II. ИССЛЕДОВАНИЕ точности СТАТИСТИЧЕСКИХ ВЫВОДОВ в РЕГРЕССИОННОМ АНАЛИЗЕ
    11. 1 Линейный (относительно оцениваемых параметров) нормальный вариант идеализированной схемы регрессионной зависимости
    11.1.2. Решение основных задач по оценке точности регрессионной модели.
    11.1.3. Случаи линейной (по предикторным переменным) и полиномиальной регрессии.
    11.2. Нелинейный нормальный вариант идеализированной схемы регрессионной зависимости
    11.2.2. Решение основных задач по оценке точности нелинейной регрессионной модели.
    11.3. Исследование точности регрессионной модели в реалистической ситуации
    ВЫВОДЫ
    Глава 12. СТАТИСТИЧЕСКИЙ АНАЛИЗ АВТОРЕГРЕССИОННЫХ ДИНАМИЧЕСКИХ ЗАВИСИМОСТЕЙ
    12.1. Дискретные динамические модели
    12.2. Авторегрессия первого порядка
    12.3. Авторегрессия произвольного порядка
    ВЫВОДЫ
    Раздел III. ИССЛЕДОВАНИЕ ЗАВИСИМОСТИ КОЛИЧЕСТВЕННОГО РЕЗУЛЬТИРУЮЩЕГО ПОКАЗАТЕЛЯ ОТ ОБЪЯСНЯЮЩИХ ПЕРЕМЕННЫХ СМЕШАННОЙ ПРИРОДЫ
    Глава 13. ДИСПЕРСИОННЫЙ И КОВАРИАЦИОННЫЙ АНАЛИЗ
    13.1. Классификация моделей дисперсионного анализа по способу организации исходных данных
    13. 2. Однофакторный дисперсионный анализ
    13.3. Полный двухфакторный дисперсионный анализ
    13.4. Модели дисперсионного анализа со случайными факторами
    13.5. Ковариационный анализ (КА) и проблема статистического исследования смесей многомерных распределений
    13.6. Влияние нарушений основных предположений
    ВЫВОДЫ
    Раздел IV. СИСТЕМЫ ОДНОВРЕМЕННЫХ УРАВНЕНИЙ И ПРОГРАММНОЕ ОБЕСПЕЧЕНИЕ АППАРАТА СТАТИСТИЧЕСКОГО ИССЛЕДОВАНИЯ ЗАВИСИМОСТЕЙ
    14.1. Системы одновременных уравнений
    14.2. Спецификация модели и проблема идентифицируемости
    14.3. Рекурсивные системы
    14.4. Двух- и трехшаговый методы наименьших квадратов
    14.5. Метод неподвижной точки
    14.6. Сравнение методов
    ВЫВОДЫ
    Глава 15. ПРОГРАММНОЕ ОБЕСПЕЧЕНИЕ СТАТИСТИЧЕСКОГО ИССЛЕДОВАНИЯ ЗАВИСИМОСТЕЙ
    ПРИЛОЖЕНИЯ. МАТЕМАТИКО-СТАТИСТИЧЕСКИЕ ТАБЛИЦЫ
    Таблица П.1. Значения функции плотности стандартного нормального закона распределения
    Таблица П.2. Значения функции стандартного нормального распределения
    ИСПОЛЬЗУЕМЫЕ В КНИГЕ ОБОЗНАЧЕНИЯ
    СПИСОК ЛИТЕРАТУРЫ

    Стандартная таблица нормального распределения

    изображения / normal-dist. js

    Это колоколообразная кривая стандартного нормального распределения.
    Это нормальное распределение со средним значением 0 и стандартным отклонением 1.

    Показывает процент населения:

    • между 0 и Z (опция «0 до Z»)
    • меньше Z (опция «До Z»)
    • больше, чем Z (опция «Z и далее»)

    Отображает только значения до 0,01%

    Вы также можете использовать приведенную ниже таблицу. В таблице показана область от 0 до Z.

    Вместо одной ДЛИННОЙ таблицы мы поместили « 0.1 » вниз, а затем « 0.01 «. (Пример использования ниже)

    З 0,00 0,01 0,02 0,03 0,04 0,05 0,06 0,07 0,08 0,09
    0,0 0,0000 0,0040 0,0080 0,0120 0,0160 0,0199 0,0239 0,0279 0,0319 0,0359
    0,1 0,0398 0,0438 0,0478 0,0517 0,0557 0,0596 0,0636 0,0675 0,0714 0,0753
    0,2 0,0793 0,0832 0,0871 0,0910 0,0948 0,0987 0,1026 0,1064 0,1103 0,1141
    0,3 0,1179 0,1217 0,1255 0,1293 0,1331 0,1368 0,1406 0,1443 0,1480 0,1517
    0,4 0,1554 0,1591 0,1628 0,1664 0,1700 0,1736 0,1772 0,1808 0,1844 0,1879
    0,5 0,1915 0,1950 0,1985 0,2019 0,2054 0,2088 0,2123 0,2157 0,2190 0,2224
    0,6 0,2257 0,2291 0,2324 0,2357 0,2389 0,2422 0,2454 0,2486 0,2517 0,2549
    0,7 0,2580 0,2611 0,2642 0,2673 0,2704 0,2734 0,2764 0,2794 0,2823 0,2852
    0,8 0,2881 0,2910 0,2939 0,2967 0,2995 0,3023 0,3051 0,3078 0,3106 0,3133
    0,9 0,3159 0,3186 0,3212 0,3238 0,3264 0,3289 0,3315 0,3340 0,3365 0,3389
    1,0 0,3413 0,3438 0,3461 0,3485 0,3508 0,3531 0,3554 0,3577 0,3599 0,3621
    1. 1 0,3643 0,3665 0,3686 0,3708 0,3729 0,3749 0,3770 0,3790 0,3810 0,3830
    1,2 0,3849 0,3869 0,3888 0,3907 0,3925 0,3944 0,3962 0,3980 0,3997 0,4015
    1,3 0,4032 0,4049 0,4066 0,4082 0,4099 0,4115 0,4131 0,4147 0,4162 0,4177
    1,4 0,4192 0,4207 0,4222 0,4236 0,4251 0,4265 0,4279 0,4292 0,4306 0,4319
    1,5 0,4332 0,4345 0,4357 0,4370 0,4382 0,4394 0,4406 0,4418 0,4429 0,4441
    1,6 0,4452 0,4463 0,4474 0,4484 0,4495 0,4505 0,4515 0,4525 0,4535 0,4545
    1,7 0,4554 0,4564 0,4573 0,4582 0,4591 0,4599 0,4608 0,4616 0,4625 0,4633
    1,8 0,4641 0,4649 0,4656 0,4664 0,4671 0,4678 0,4686 0,4693 0,4699 0,4706
    1,9 0,4713 0,4719 0,4726 0,4732 0,4738 0,4744 0,4750 0,4756 0,4761 0,4767
    2,0 0,4772 0,4778 0,4783 0,4788 0,4793 0,4798 0,4803 0,4808 0,4812 0,4817
    2. 1 0,4821 0,4826 0,4830 0,4834 0,4838 0,4842 0,4846 0,4850 0,4854 0,4857
    2.2 0,4861 0,4864 0,4868 0,4871 0,4875 0,4878 0,4881 0,4884 0,4887 0,4890
    2,3 0,4893 0,4896 0,4898 0,4901 0,4904 0,4906 0,4909 0,4911 0,4913 0,4916
    2,4 0,4918 0,4920 0,4922 0,4925 0,4927 0,4929 0,4931 0,4932 0,4934 0,4936
    2,5 0,4938 0,4940 0,4941 0,4943 0,4945 0,4946 0,4948 0,4949 0,4951 0,4952
    2,6 0,4953 0,4955 0,4956 0,4957 0,4959 0,4960 0,4961 0,4962 0,4963 0,4964
    2,7 0,4965 0,4966 0,4967 0,4968 0,4969 0,4970 0,4971 0,4972 0,4973 0,4974
    2,8 0,4974 0,4975 0,4976 0,4977 0,4977 0,4978 0,4979 0,4979 0,4980 0,4981
    2,9 0,4981 0,4982 0,4982 0,4983 0,4984 0,4984 0,4985 0,4985 0,4986 0,4986
    3,0 0,4987 0,4987 0,4987 0,4988 0,4988 0,4989 0,4989 0,4989 0,4990 0,4990

     

    Пример: Процент населения от 0 до 0,45

    Начните со строки 0,4 и читайте дальше до 0,45: есть значение 0,1736

    А 0,1736 — это 17,36%

    Итак, 17,36% населения находятся в диапазоне от 0 до 0,45 стандартных отклонений от среднего.

    Поскольку кривая симметрична, одну и ту же таблицу можно использовать для значений, идущих в любом направлении, поэтому отрицательный 0,45 также имеет площадь 0,1736

    Пример: Процент населения Z Между −1 и 2

    От от −1 до 0 то же, что и от 0 до +1 :

    В строке для 1,0, первый столбец 1,00, есть значение 0,3413

    От 0 до +2 :

    В строке для 2,0, первый столбец 2,00, есть значение 0,4772

    Добавьте два, чтобы получить сумму от -1 до 2:

    0,3413 + 0,4772 = 0,8185

    И 0,8185 9002 3 равно 81,85%

    Таким образом, 81,85% населения находятся между -1 и +2 стандартных отклонения от среднего.

     

    1486, 1487, 1488, 1489, 1490, 1491, 3846, 3847, 3848, 3849

    Квантиль — это ключ к пониманию распределения вероятностей

    Если вы когда-нибудь чувствовали себя запутанными при использовании распределения вероятностей, эта статья для вас.

    Agnieszka Kujawska, PhD

    ·

    Читать

    Опубликовано в

    ·

    13 мин чтения

    ·

    Июн 26, 2021

    Photo by Joshua Earle on Unsplash

    Вы много раз встречались с распределением вероятностей. Вы знаете, что есть несколько разных типов. Но в глубине души вы чувствуете смущение, когда вам нужно использовать его на практике. В чем, черт возьми, разница между распределением вероятностей и кумулятивным распределением вероятностей? Должен ли я проверять уровень достоверности или альфу по оси X или Y? Если да, то эта статья для вас. В конце концов, вы будете чувствовать себя комфортно, используя распределения вероятностей для дискретных или непрерывных случайных величин. Давайте погрузимся в это!

    В этой статье мы рассмотрим следующие темы:

    1. Функция плотности вероятности (PDF)
    2. Функция массы вероятности (PMF)
    3. Кумулятивное распределение вероятностей (CDF)
      3. 1 Кумулятивное распределение вероятностей для ДИСКРЕТНЫХ случайных величин (CMF) 9000 5 3.2 Кумулятивное распределение вероятностей для НЕПРЕРЫВНЫХ случайных величин (CDF)
    4. Сводка распределений вероятностей
    5. Функция квантилей
    6. Спасибо за чтение и ссылки

    Распределение плотности вероятности нормального распределения — это то, о чем люди чаще всего думают, когда слышат слово «распределение». Он имеет специфическую форму колокола:

    PDF стандартного нормального распределения (нулевое среднее и стандартное отклонение 1). Источник: изображение автора

    Функция плотности вероятности (PDF) сопоставляет значение с его плотностью вероятности [1]. Это понятие похоже на физику, где плотность вещества — это его масса на единицу объема. Например, 1 литр воды весит примерно 1 кг, поэтому плотность воды составляет примерно 1 кг/л или 1000 кг/м³. Аналогично, плотность вероятности измеряет вероятность на единицу х .

    PDF относится к непрерывной случайной величине , что означает, что переменная может принимать любое значение в пределах определенного диапазона действительных чисел. Random показывает неопределенность того, какие значения может принимать переменная. Это дает бесконечное количество возможностей, например 0,1, но также и 0,101, 0,1001 и т. д. Таким образом, вероятность того, что непрерывная случайная величина будет равна заданному значению, равна нулю.

    Вероятность на графике PDF представлена ​​площадью под кривой плотности. Площадь под точкой равна нулю. Вот почему PDF используется для проверки вероятности того, что случайная величина попадает в заданный диапазон значений, а не для принятия какого-либо конкретного значения. Например, какова вероятность того, что мы потеряем деньги, инвестируя в фонд так, что доходность будет отрицательной? Здесь мы рассматриваем все доходы меньше нуля.

    Интуитивно PDF представляет собой линию, описывающую гистограмму. Например, мы хотим разделить 992 участника эксперимента на возрастные группы (0–10, 11–20 и т. д.). Мы подсчитываем, сколько участников попадает в каждую группу, и представляем это в виде столбцов на гистограмме:

    Гистограмма из 992 участников, разделенных на возрастные группы. Данные, полученные из нормального распределения. Изображение автора.

    Насколько высока вероятность того, что человек, которого мы случайно выберем, будет членом данной возрастной группы? Во-первых, мы должны преобразовать распределение частот в распределение вероятностей. Это означает вычисление плотности вероятности на основе количества участников в каждой группе. Так как бары имеют прямоугольную форму и площадь под функцией плотности вероятности всегда равна 1, мы можем использовать упрощенное уравнение:

    Для частот, представленных на предыдущем графике, мы имеем:

    Теперь мы можем построить наши данные, используя плотности вместо подсчетов по оси Y. Красная кривая соединяет расчетные точки и обозначает функцию плотности вероятности:

    PDF-график автора. Красная линия — функция плотности вероятности.

    Но обратите внимание, что я сгенерировал данные для этого графика из нормального распределения. Вот почему PDF и гистограмма так хорошо подходят. PDF имеет «закрытую» форму, что требует предварительного определения распределения и параметров (среднее значение и стандартное отклонение в случае нормального распределения). Гистограмма использует необработанные данные, поэтому она показывает реальное распределение. Это позволяет обнаруживать аномалии, особенно при большом количестве баров.

    Интересуют другие параметры, используемые для описания распределения (математическое ожидание, дисперсия, асимметрия и эксцесс)? Перейти сюда:

    Статистические моменты в интервью по науке о данных

    Основная математика для специалистов по данным, объясненная с нуля

    в направлении datascience.com

    Ключевые моменты, которые следует помнить из приведенного выше анализа:

    • Вероятность — это площадь под вероятностью кривая плотности (PDF).
    • Вероятность того, что непрерывная случайная величина примет заданное значение, равна нулю. Итак, для заданного значения x мы можем проверить только плотность вероятности, что не очень полезно.
    • Поэтому мы ориентируемся на интервалы значений. Это позволяет нам делать вероятностные утверждения о диапазоне значений. Например, есть вероятность 50%, что участнику будет не менее 40 лет.

    Функция массы вероятности (PMF) относится к дискретным случайным величинам. В отличие от непрерывных случайных величин, дискретные случайные величины могут принимать только счетное число дискретных значений, таких как 0, 1, 2,…. Простыми примерами являются бросание игральной кости, подбрасывание монеты или обнаружение мошеннических транзакций (мошенничество либо есть, либо его нет).

    Подобно непрерывным случайным величинам, мы можем создать гистограмму дискретных данных. Но нет необходимости агрегировать значения в интервалы. Рассмотрим сумму бросков пары игральных костей. Количество результатов конечно, так как значения на обоих кубиках от 1 до 6. На графике ниже показан пример гистограммы для 1000 бросков правильной пары игральных костей:

    Гистограмма сумм при 1000-кратном броске правильной пары игральных костей . Изображение автора.

    Оба кубика являются правильными, что означает, что вероятность выпадения каждого числа от 1 до 6 одинакова, равная 1/6. Таким образом, самая популярная сумма равна 7. Как и в случае с непрерывными случайными величинами, мы можем выразить каждый результат как вероятность.

    Если мы бросим пару кубиков, возможны 36 исходов (по 6 вариантов на каждом кубике). Если сумма равна 2, возможна только одна комбинация: (1,1). Таким образом, вероятность получить сумму, равную 2, равна 1/36 = 0,0278. Аналогично для суммы 12, возможно только для (6,6). Точно так же мы можем рассчитать вероятности других возможных исходов. Результаты, представленные на графике, создают функцию массы вероятности (PMF):

    PMF суммы справедливой пары игральных костей. Изображение автора.

    Подводя итог, мы рассмотрели следующие типы графиков:

    • Гистограмма — это график, показывающий, сколько раз каждый диапазон значений появляется в наборе данных. Он не требует каких-либо предположений о распределении, но мы должны заранее указать количество баров. Гистограмма строится из конечного числа выборок. Сумма значений гистограммы для всех баров равна общему количеству выборок.
    • Функция плотности вероятности (PDF) описывает плотность вероятности непрерывных случайных величин . Вероятность на PDF представляет собой площадь под кривой плотности. Поскольку вероятность данного значения равна нулю для непрерывных случайных величин, PDF используется для проверки вероятности того, что переменная попадает в заданный интервал. Вся площадь под PDF равна единице.
    • Функция массы вероятности (PMF) описывает вероятность дискретных случайных величин . Это означает, что переменная может принимать только счетное число дискретных значений, таких как 0, 1, 2 и т. д. Сумма вероятностей всех дискретных значений в PMF равна единице.

    Хотя все они очень полезны и широко используются в отрасли, есть еще одно важное распределение вероятностей — кумулятивная функция распределения (CDF).

    Кумулятивная функция распределения (CDF) случайной величины X описывает вероятность (шансы) того, что X примет значение, равное или меньшее x. Математически это можно выразить так:

    3.1. Кумулятивная функция распределения ДИСКРЕТНОГО распределения вероятностей (CDF или CMF)

    Взяв предыдущий пример с подбрасыванием правильной пары игральных костей, мы можем спросить: какова вероятность того, что сумма двух игральных костей меньше или равна 3? Нам нужно добавить вероятность суммы, равной 2 (0,0278), и вероятность суммы 3 (0,0556), поэтому совокупная вероятность для x = 3 составляет 0,0278 + 0,0556 = 0,0834. Затем мы повторяем процесс добавления для каждого дискретного значения, чтобы получить кумулятивную функцию распределения дискретного распределения вероятностей:

    Кумулятивную функцию вероятности дискретного распределения вероятностей. Изображение автора.

    Как видно на графике, кумулятивная функция вероятности для максимально возможного исхода равна 1. Поскольку сумма двух игральных костей может принимать только целые значения, график можно выразить с помощью столбцов:

    Накопительная функция вероятности дискретное распределение вероятностей. Изображение автора.

    3.2. Кумулятивная функция распределения НЕПРЕРЫВНОГО распределения вероятностей (CDF)

    Идея CDF для непрерывных переменных такая же, как и для дискретных переменных. Ось Y показывает вероятность того, что X примет значения, равные или меньшие, чем x. Отличие в том, что вероятность меняется даже при небольших перемещениях по оси x.
    В примере с групповым возрастом участников кумулятивная функция распределения выглядит следующим образом:

    Кумулятивная функция распределения непрерывного распределения вероятностей (CDF). Изображение автора.

    Графики ниже сравнивают PDF и CDF нормального распределения с нулевым средним и стандартным отклонением, равным единице:

    PDF и CDF нормального распределения N(0,1). Изображение автора.

    Мы можем сделать вывод, что:

    • CDF является неубывающей функцией. Он показывает вероятность того, что переменная равна или меньше x, поэтому она может увеличиваться только с увеличением значения x.
    • Мы можем проверить вероятность по обоим графикам, но использование CDF более просто. CDF показывает вероятность по оси y, а PDF имеет плотность вероятности по оси y. В случае PDF вероятность представляет собой площадь под кривой PDF.
    • Поскольку нормальное распределение симметрично, CDF при x=0 (что означает среднее значение) составляет 0,5.
    • Функция CDF с левой стороны асимптотична к 0 и 1 с правой стороны графика. Точные значения x зависят от типа распределения и параметров (среднее значение и стандартное отклонение для нормального распределения).

    До сих пор мы рассмотрели три способа описания распределения вероятностей: функция плотности вероятности (PDF), функция массы вероятности (PMF) и кумулятивная функция распределения (CDF). Основные различия между PDF и PMF представлены в таблице ниже:

    Основные различия между PDF и PMF. Изображение автора.

    Кумулятивная функция распределения показывает вероятность того, что X примет максимальное значение x. Он суммирует шансы для всех меньших значений и шансов, равных x. Поскольку ось Y представляет собой вероятность, использование CDF часто более просто, чем для PDF.

    На следующей схеме показаны типичные графики каждого распределения, по часовой стрелке и начиная с верхнего левого угла: PDF, PMF, CMF, CDF. Он обобщает высокоуровневую характеристику и описывает отношения между заданными типами функций распределения.

    Сравнение различных типов дистрибутивов. Изображение автора вдохновлено [1,2].

    Как видно выше, существует некоторая связь между различными способами отображения распределения вероятностей.

    • Для непрерывных случайных величин мы можем легко построить PDF и CDF. Область под PDF — это вероятность, поэтому нам нужно интегрировать, чтобы преобразовать PDF в CDF, или дифференцировать, чтобы перейти от CDF к PDF.
    • Для дискретных случайных величин PMF показывает вероятность, а CDF (CMF) — кумулятивную вероятность. Чтобы получить CMF из PMF, мы должны сложить вероятности до заданного x. Чтобы пойти наоборот (от CMF к PMF), мы должны вычислить разницу между шагами.
    • Если мы разделим все значения на набор бинов (см. примеры с гистограммами выше), мы можем перейти от PDF к виду PMF. Он использует диапазон значений/интервалов и может рассматриваться как аппроксимация PDF. Чтобы перейти от дискретного кумулятивного распределения к непрерывной функции, необходима некоторая форма сглаживания. Это можно сделать, предположив, что данные поступают из определенного непрерывного распределения, такого как нормальное или экспоненциальное, и оценив параметры этого распределения. Изменение дискретной и непрерывной случайной величины в обоих направлениях следует рассматривать как аппроксимацию.

    Позвольте представить суперзвезду распределений — функцию квантилей. Это позволяет использовать распределения для многих практических целей, таких как поиск доверительных интервалов и проверка гипотез.

    Математическое определение состоит в том, что квантильная функция является обратной функцией распределения при α. Он определяет значение случайной величины так, что вероятность того, что переменная меньше или равна этому значению, равна заданной вероятности:

    Где F⁻¹(α) обозначает α-квантиль X.

    Сейчас это может показаться немного загадочным, но при ближайшем рассмотрении сомнения развеются. Предположим, что мы хотим проверить 5% общей площади в нижнем хвосте распределения. Мы называем это нижним 5% квантилем X и записываем как F⁻¹(0,05). Квантиль — это распределение вероятностей, разделенное на области с равной вероятностью. Если рассматривать проценты, то сначала делим раздачу на 100 штук. Когда мы смотрим в PDF, 5-й квантиль — это точка, которая отсекает площадь 5% в нижней части распределения:

    Нижний 5% квантиль для нормального распределения N(0,1). Изображение автора.

    Площадь под PDF слева от красной линии составляет ровно 5% от общей площади под кривой. Это подразумевает вероятность 5%. Первым шагом к рисованию красной линии было вычисление, где заканчивается 0,05 общей площади (здесь x=-1,645). Это можно сделать с помощью программного обеспечения (например, функция qnorm() в R или scipy.stats.norm.ppf() в Python) или вручную с использованием z-таблиц (пример здесь).

    Поскольку CDF имеет вероятность (α) по оси y, проще найти это значение здесь:

    Нижний 5% квантиль для нормального распределения N(0,1). Изображение автора.

    Это показывает, насколько полезны графики CDF. Мы можем использовать CDF в обоих направлениях:

    • Если у нас есть значение z (или значение x, значение на оси x), мы можем проверить вероятность того, что X примет значение, равное или меньшее, чем x. Например, какова вероятность того, что средняя продолжительность пребывания клиента в интернет-магазине составляет полчаса или меньше?
    • Если у нас есть вероятность, мы можем проверить значение, которое отсекает область данной альфы. Например, с 90% уверенности, можно сказать, что клиент проводит в интернет-магазинах не менее X часов.

    В приведенном выше примере мы рассмотрели только односторонний 5% квантиль (нижний хвост). Мы можем сделать то же самое для 5% вероятности с двух сторон. Это означает, что мы ищем 5% общей площади под PDF, но разделены на 2,5% нижнего квантиля (слева) и 2,5% верхнего квантиля (на правой стороне графика).

    Двусторонняя вероятность 5%. Изображение автора.

    Таким образом, квантили являются прямой связью между этими графиками.

    Основываясь на графиках, мы можем сказать, что у нас есть 95% уверенность в том, что истинный параметр (среднее значение) находится между -1,96 и 1,96. Или что существует 5% вероятность того, что оно находится за пределами диапазона от -1,96 до 1,96.

    Приведенная выше интерпретация подчеркивает, что:

    • уровень достоверности говорит нам, насколько вероятно рассматриваемое событие или каковы шансы того, что данный параметр находится в заданном диапазоне значений.
    • альфа или уровень значимости — это вероятность. Мы можем проверить это по оси Y на графике CDF. Альфа — это один минус уровень достоверности.

    Несколько замечаний:

    • Обратная функция Φ⁻¹(α) является α-квантилем
    • Когда α мало, квантиль также называется критическим значением 0010 Некоторые квантили имеют специальные имена . Если мы разделим вероятность на 100 частей, мы получим процентили. Мы можем сказать 5-й процентиль вместо 5% квантиля. 4-квантили называются квартилями и делятся на 4 части с разбивкой по значениям 25%, 50% (медиана) и 75%.
    • Для стандартного нормального распределения (нормальное распределение с нулевым средним значением и стандартным отклонением, равным единице N(0,1)), которое симметрично относительно нуля, мы имеем:

    Это доказано на графиках выше, поскольку мы получаем — 1,96 на нижнем оперении и 1,96 на верхнем оперении.

    Используя квантили, PDF, CDF, мы можем ответить на разные вопросы в зависимости от информации, которой мы владеем, например:

    • Учитывая выборочное среднее, каков диапазон значений, содержащих среднее значение генеральной совокупности, в котором мы достаточно уверены? «Разумно» может принимать различные процентные значения и зависит от цели нашего исследования.
    • С какой степенью уверенности можно сказать, что доходность не будет отрицательной?

    Я рад, что вы дочитали до конца этой статьи. Мы рассмотрели различные типы распределений вероятностей: функция плотности вероятности (PDF), функция массы вероятности (PMF) и кумулятивная функция плотности (CDF). Затем мы обсудили функцию количества. Он связывает различные способы описания дистрибутивов (PDF и CDF) и позволяет нам использовать эти дистрибутивы очень практичным образом. Надеюсь, это было увлекательное путешествие для вас.

    Помните, что самый эффективный способ выучить (математические) навыки — это практика . Так что не ждите, пока вы почувствуете себя «готовым», просто возьмите ручку и бумагу (или ваше любимое программное обеспечение) и попробуйте несколько примеров самостоятельно. Держу за тебя пальцы скрещенными.

    Я буду рад услышать ваши мысли и вопросы в разделе комментариев ниже, связавшись со мной напрямую через мой профиль LinkedIn или по телефону [email protected]. До скорой встречи!

    Вам также может понравиться:

    Statistical Moments in Data Science интервью

    Основная математика для специалистов по данным: объяснение с нуля

    в направлении datascience.

    Производная: еще пара-тройка глупых вопросов: marta_inj — LiveJournal

    ?
    Categories:
    • Наука
    • Образование
    • Cancel

    Все знают, что на ноль делить нельзя. Но почему?
    Потому что не имеет смысла – ответствуют нам школьные учителя.
    Хотя вроде бы смысл есть. Любое число, деленное на ноль, должно быть равно бесконечности.
    А если ноль поделить на ноль? Единице, наверное.
    Вроде логично…

    Но тут выплывает производная.

    Производной функции y = f(x) в точке x называется предел (если он существует и конечен) отношения приращения функции к приращению аргумента при условии, что последнее стремится к нулю.

    Ага, здесь у нас как раз знаменатель стремится к нулю, но сама функция почему-то не к бесконечности стремится, а к другим значениям.

    Например, известно, что производная константы (например, числа) равна нулю. Но что является функцией, а что аргументом в этом случае?..

    Производная указывает нам на скорость изменения функции- так написано во многих учебниках. Число вроде бы не меняется, значит, производная равна нулю… Но всплывает вопрос – а с чего мы взяли, что число не меняется? Потому что это аксиома?
    Если взять единицу длины – в виде палочки и веревочки – можем мы эту единицу длины изменить? Можем растянуть? Можем. Можем сжать? Можем. Выходит, на физическом плане неизменность числа никак не гарантируется, она – эта неизменность — существует лишь теоретически, в виртуальных математических построениях.

    А вот производная икс квадрат – это два икс. Почему?!

    Вот пример (из лекций по физике Цаплина):

    Путь s — скалярная величина, равная полной длине отрезка траектории, пройденной МТ за время движения.

    Уравнение пути

    s = s(t) .

    Перемещение — вектор, проведенный из начального положения (точка А) в конечное (точка В),

    Численные значения и в случае прямолинейного движения совпадают. В случае же криволинейного движения они совпадают только в пределе, т.е. для бесконечно малого перемещения
    .

    Здесь нападает ступор… В пределе совпадают, а по факту нет. Что это значит? Что в микромире путь и перемещение совпадают? А в макромире – нет?..

    Интересное из Википедии:
    • Определение через колебания: функция непрерывна в точке, если её колебание в данной точке равно нулю.
    …..
    Если функция имеет разрыв в данной точке (то есть предел функции в данной точке отсутствует или не совпадает со значением функции в данной точке), то для числовых функций возникает два возможных варианта, связанных с существованием у числовых функций односторонних пределов:
    • если оба односторонних предела существуют и конечны, то такую точку называют точкой разрыва первого рода. К точкам разрыва первого рода относят устранимые разрывы и скачки.
    • если хотя бы один из односторонних пределов не существует или не является конечной величиной, то такую точку называют точкой разрыва второго рода. К точкам разрыва второго рода относят полюса и точки существенного разрыва.

    Tags: Вопросы к науке, Смысл математики

    Subscribe

    • Еще раз про свойства сознания

      Для того, чтобы мы могли найти отличия на картинках, обе эти картинки должны быть рядом в одном поле зрения. Если мы попробуем расположить эти…

    • «Потерянные» участки тела

      Описываю гипотезу, почему мышечный зажим одновременно ускользает от сознания и захватывает соседние и даже отдаленные мышцы, создавая напряжение в…

    • Поговорим про мышечный зажим

      Мышечный зажим — это хронически напряженная мышца, которую мы не замечаем. Не замечаем потому, что её состояние не меняется, а значит, из сферы…

    Photo

    Hint http://pics. livejournal.com/igrick/pic/000r1edq

    2 = \displaystyle {\underbrace {x+x+x+ \ldots +x}_{x \ times}}$

    Теперь пусть

    $ f(x) = \displaystyle {\underbrace {x+x+x+ \ ldots +x}_{x \ times}}$

    затем

    $ f'(x) = \dfrac{d}{dx} \left( \displaystyle {\underbrace {x+x+x+ \ldots + x} _ {x \ times}} \right) $

    $ f ‘(x) = \ displaystyle {\ underbrace {\ dfrac {d} {dx} x + \ dfrac {d} {dx} x + \ ldots + \dfrac{d}{dx} x}_{x \ times}}$

    $ f'(x)=\displaystyle {\underbrace {1 + 1 + \ldots + 1}_{x \ times}} $ 92 = х[х]+х\{х\}$

  • $d/dx {[x²]}= d/dx \left( {x[x] +x \{x\} }\right)$
  • (дифференциация по частям)
  • $= 1\cdot [x]+x \cdot [x]’+ 1\cdot \{x\} + x \cdot \{x\}’$
  • , так как $d/dx (x)=x’=1$ и [x]’ & {x}’ представляют дифференцирование каждого по x.
  • $=[x]+\{x\}+x \left({[x]’+\{x\}’ }\right)$
  • $=x+x (x’)$
  • $=х+х=2х$
    • Yesmanapple прислал свое мнение об этой статье. Взгляни.
    • wnoise предложил эту ссылку:
    • Умножение не повторяется Сложение.
    • Функция наибольшего целого числа

    Предыдущий пост: Решение головоломной задачи Рамануджана

    Следующий пост: Формула Вейна и законы Вейна

    Видео-вопрос: Дифференцирование функции по отношению к другой функции с использованием параметрического дифференцирования

    Используя параметрическое дифференцирование, определите производную от 5𝑥³ + 𝑥² − 2 относительно 4𝑥² + 8.

    Стенограмма видео

    Используя параметрическое дифференцирование, определите производную пяти 𝑥 в кубе плюс 𝑥 в квадрате минус два относительно четырех 𝑥 в квадрате плюс восемь.

    Напомним, что если бы у нас было 𝑦 равно некоторой функции 𝑓 от 𝑥 и 𝑧 равно некоторой функции 𝑔 от 𝑥, то по цепному правилу. Производная от 𝑦 по 𝑧 равна производной от 𝑦 по 𝑥, умноженной на производную от 𝑥 по 𝑧. Затем, установив 𝑥 равным обратному 𝑔 𝑧, мы получим, что производная 𝑥 по 𝑧 равна единице, деленной на производную 𝑧 по 𝑥, используя нашу теорему об обратной функции. Используя это, мы можем вычислить производную от 𝑦 по 𝑧, сначала вычислив производную от 𝑦 по 𝑥, а затем разделив ее на производную от 𝑧 по 𝑥.

    Вопрос требует, чтобы мы вычислили производную пяти 𝑥 в кубе плюс 𝑥 в квадрате минус два. И он хочет, чтобы мы сделали это относительно четырех 𝑥 в квадрате плюс восемь. Итак, если мы установим 𝑦 равным пяти 𝑥 в кубе плюс 𝑥 в квадрате минус два и 𝑧 равным четырем 𝑥 в квадрате плюс восемь, то d𝑦 d𝑧 — это то, что вопрос хочет, чтобы мы вычислили. Это производная от пяти 𝑥 в кубе плюс 𝑥 в квадрате минус два относительно четырех 𝑥 в квадрате плюс восемь. И мы имеем, что это равно производной от 𝑦 по 𝑥, деленной на производную от 𝑧 по 𝑥.

    Теперь мы можем вычислить и то, и другое. Производная от 𝑦 по 𝑥 равна производной пяти 𝑥 в кубе плюс 𝑥 в квадрате минус два по 𝑥. Мы можем дифференцировать этот термин за термином. Чтобы дифференцировать пять 𝑥 в кубе, мы умножаем его на показатель степени, а затем уменьшаем показатель степени на единицу. Это дает нам 15𝑥 в квадрате. Мы делаем то же самое, чтобы дифференцировать 𝑥 в квадрате. Мы умножаем на показатель степени двух, а затем уменьшаем показатель степени на единицу, что дает нам два 𝑥.

    Наконец, производная любой константы просто равна нулю. Таким образом, производная от отрицательных двух просто равна нулю. Итак, мы показали, что d𝑦 d𝑥 равно 15𝑥 в квадрате плюс два 𝑥. Мы можем сделать то же самое, чтобы вычислить производную от 𝑧 по 𝑥. Это производная от четырех 𝑥 в квадрате плюс восемь относительно 𝑥.

    Чтобы дифференцировать четыре 𝑥 в квадрате, мы умножаем на показатель степени двух, а затем уменьшаем показатель степени на единицу, что дает нам восемь 𝑥. А затем, чтобы продифференцировать постоянную восемь, мы просто получаем ноль, что дает нам, что производная от 𝑧 по 𝑥 равна восьми 𝑥. Подстановка их в наше уравнение дает нам, что производная от 𝑦 по отношению к 𝑧 равна 15𝑥 в квадрате плюс два 𝑥, все деленные на восемь 𝑥.

    © 2015 - 2019 Муниципальное казённое общеобразовательное учреждение «Таловская средняя школа»

    Карта сайта